You are on page 1of 49

NEURO FORM 3 by Sergio Angulo

1 E
2 E
3 E
4 C
5 A
6 E
7 E
8 E
9 D
10 L
11 A
12 C
13 K
14 B
15 B
16 F
17 IV ANTIBIOTIC
18 C
19 A
20 C
21 G
22 C
23 E
24 Maybe E, not C
25 A
26 B
27 C
28 B
29 B
30 C
31 E
32 A
33 E
34 E
35 D
36 E
37 E
38 B
39 E
40 C
41 D
42 C
43 D
44 G
45 K
46 D
47 C
48 A
49 E
50 E

1. 42 yo woman alcoholic, severe burning paresthesias both feet, decreased vibratory sens
both ankles 2 months after ttmt for pulm TBC. Drugs include INH, rifampicin, vit B6, vit
B9. Failure to take which drug?
E. Vit B6
INH competes with vitamin B6 (pyridoxine) in its action as a cofactor in the synthesis of
synaptic neurotransmitters. Resulting dose-related neurologic side effects include
peripheral neuropathy, ataxia, and paresthesia. Such side effects are uncommon in the
absence of risk factors.
Patients at increased risk for INH-induced neurotoxicity include:
Older adults
Individuals with chronic liver disease
Individuals who are malnourished
Individuals with HIV infection
Individuals with renal failure
Diabetics
Alcoholics
Pregnant or breastfeeding women
Children
The risk of neurologic side effects can be reduced by concomitant administration of
pyridoxine (vitamin B6). The usual dose is 10 mg/day (in patients who do not have high-
risk features) and 25 to 50 mg/day (in those with one or more risk factors) [8].
Treatment of INH-induced neuropathies with pyridoxine generally requires higher doses
(100 to 200 mg/day).
2. 42 yo woman, confused state for possible 3 days. Hx lithium, haloperidol, clonazepam
for psych condition. 99.5F, pulse 85/min, BP 140/90. Pupils reactive 2mm. Coarse
tremor upper extremities. Increased deep tendon reflexes. Responds to vigorous stim.
Speech slow slurred. Appears sleepy and perplexed. WBC 14000, Urea 60mg/dl, Creat
2.5mg/dl, lithium 3.5 (normal 0.6-1.2). In add to discontinue lithium, next step?
E. Begin hemodialysis
Lithium is excreted almost entirely by the kidneys and is handled in a manner similar to
sodium. Lithium is freely filtered but over 60 percent is then reabsorbed by the proximal
tubules. Volume depletion or renal impairment from any cause increases lithium
reabsorption. Examples of such conditions include gastrointestinal losses, acute
decompensated heart failure, cirrhosis, and the administration of diuretics, nonsteroidal
antiinflammatory drugs, or angiotensin converting enzyme inhibitors. With lithium
poisoning, it is important to ask about dehydration, medications, and coingestants. Any
condition causing dehydration, such as vomiting or diarrhea, fever or heat-related
illness, anorexia or infection, may contribute to lithium toxicity.
Neurologic findings develop late in acute lithium poisoning because time is required for
the drug to be absorbed and to penetrate the central nervous system (CNS). Potential
neurologic symptoms and signs include sluggishness, ataxia, confusion or agitation, and
neuromuscular excitability, which can manifest as irregular coarse tremors,
fasciculations, or myoclonic jerks. Severe lithium intoxication can lead to seizures,
nonconvulsive status epilepticus, and encephalopathy.
The treatment for lithium poisoning includes intravenous fluids to maintain the
glomerular filtration rate and replace losses, gastrointestinal decontamination in
selected circumstances (Whole bowel irrigation with polyethylene glycol (PEG), and
hemodialysis in cases of severe toxicity.
Perform hemodialysis for the following indications:
Serum lithium concentration is greater than 4 mEq/L (or mmol/L), regardless of
the clinical status of the patient.
Serum lithium concentration is greater than 2.5 mEq/L (or mmol/L) and the
patient manifests signs of significant lithium toxicity (eg, seizures, depressed
mental status), has renal insufficiency or other conditions that limit lithium
excretion, or suffers from an illness that would be exacerbated by aggressive IV
fluid hydration (eg, heart failure)
3. 47 yo man, alcoholic, ER for confusion. Lives alone, disoriented for 4 hours. Empty
container with sweet odor in apt. Creat 1.5mEq/l, HCO3 8mEq/l. ketones (-). Urianalysis,
crystals. Cause?
F. Ethylene glycol
Methanol and ethylene glycol are frequently found in high concentration in automotive
antifreeze and de-icing solutions, windshield wiper fluid, solvents, cleaners, fuels, and
other industrial products. Few conditions other than methanol and ethylene glycol
intoxication cause a profound high anion gap metabolic acidosis (serum bicarbonate less
than 8 meq/L (or 8 mmol/L)), and most of these conditions present in a characteristic
fashion with a high serum lactate (eg, status epilepticus, profound shock, ischemic
bowel) or diabetic ketoacidosis (table 2). Ethylene glycol metabolites target the kidney
and lead to reversible oliguric or anuric acute kidney injury (acute renal failure), which in
turn slows elimination of ethylene glycol [7]. The renal failure is primarily due to
glycolate-induced damage to tubules, although tubule obstruction from precipitated
oxalate crystals may contribute [8,9]. Hypocalcemia in ethylene glycol overdose results
from calcium oxalate formation. With ingestions of either parent alcohol, a profound
anion gap metabolic acidosis develops, which directly correlates with the accumulation
of toxic acid metabolites [2,4]. Acidemia increases the ability of the toxic metabolites to
penetrate cells, further depressing CNS function and causing a rapid downward spiral of
hypoxia and acidemia [10].
Urine testing — Examination of the urine for oxalate crystals and fluorescence is
frequently performed in patients with possible ethylene glycol poisoning, but care
should be taken not to over-interpret positive or negative results.
4. 27 yo man. Hx 2w neck and left shoulder pain, radiating through lat surface of left arm
to thumb and index. Decreased biceps reflex. Strength 4/5 extension left wrist. Sens
light touch decreased left thumb and index. Dx?
C. C6 radiculopathy

5. 62 yo man. Hx 3y forgetfulness. Difficult remembering names. Progressive difficult with


activities daily living. No head trauma, no drugs, no serious illness. Indifferent mood and
affect. Oriented person and place. Disoriented in time. No auditory/visual halluc.
Minimental 22/30. Which ttmt?
a. Acetylcholinisterase inhibitors.
Probable AD dementia is a syndrome of dementia defined by the following
characteristics:
Interference with ability to function at work or at usual activities
A decline from a previous level of functioning and performing
Not explained by delirium or major psychiatric disorder
Cognitive impairment established by history-taking from the patient and a
knowledgeable informant; and objective bedside mental status examination or
neuropsychological testing
Cognitive impairment involving a minimum of two of the following domains:
impaired ability to acquire and remember new information
impaired reasoning and handing of complex tasks, poor judgment
impaired visuospatial abilities
impaired language functions
changes in personality, behavior or comportment
We suggest a treatment trial with a cholinesterase inhibitor for patients with mild to
moderate dementia (MMSE 10-26) (Grade 2A). The choice between donepezil,
rivastigmine, and galantamine can be based upon cost, individual patient tolerance, and
physician experience, as efficacy appears to be similar.
In patients with mild to moderate Alzheimer dementia (AD) who are interested in
seeking therapy with vitamins or who are unable or unwilling to take memantine, we
suggest supplementation with vitamin E (2000 IU daily) (Grade 2C). The benefits of
vitamin E are likely to be modest and could be offset by combination therapy with
memantine. Vitamin E is not recommended for other forms of dementia or for the
prevention of AD.
In patients with moderate to advanced dementia (MMSE <17), we suggest adding
memantine (10 mg twice daily) to a cholinesterase inhibitor, or using memantine alone
in patients who do not tolerate or benefit from a cholinesterase inhibitor (Grade 2B).
In patients with severe dementia (MMSE <10), we suggest continuing memantine,
given the possibility that memantine may be disease-modifying (Grade 2C). However, in
some patients with advanced dementia it may make sense to discontinue
administration of medications to maximize quality of life and patient comfort.

6. 77 yo man, HTN fllow up. Controlled with diuretic. BP 140/70. Deep tendon reflexes
absent in ankles. Next step management?
E. no additional management indicated.

7. 67 yo woman. ER 1h ago with episode of 25 min right facial drop, weakness right arm
hand. Hx of HTN, DM2, CAD, chronic AF, Gout. Medications metoprolol, fosinopril,
glyburide, allopurinol. Pulse 80/min irregularly irregular, resp 18/min, BP 170/100. No
murmur. Decreased sensation pinprick distal lower extremities. Achilles tendon reflex
decreased. CT no infarctation or hemorrhage. Echocard, thrombus left atrial, left vent
hypertrophy, mild mitral regurg. Carotid US, mild stenosis right and left internal carotid
art without thrombus or ulcer. Most appropriate to prevent next episode?
E. Warfarin
With the major exception of patients with atrial fibrillation, where anticoagulation is
superior to aspirin, randomized clinical trials have found no statistically significant
difference between aspirin and anticoagulant therapy for reducing the risk of recurrent
ischemic stroke [79]. However, the risk of major hemorrhage is higher with warfarin
than with aspirin.
Since antiplatelet agents are effective for the prevention of recurrent ischemic stroke in
patients with a history of noncardioembolic ischemic stroke or TIA of atherothrombotic,
lacunar (small vessel occlusive), or cryptogenic type, nearly all such patients should be
treated with an antiplatelet agent. Aspirin (50 to 100 mg daily), clopidogrel (75 mg
daily), and the combination of aspirin-extended-release dipyridamole (25 mg/200 mg
twice a day) are all acceptable options for preventing recurrent noncardioembolic
ischemic stroke.
Long-term anticoagulation with warfarin, dabigatran, apixaban, or rivaroxaban should
be considered as prevention for patients with chronic nonvalvular atrial fibrillation
who have had an ischemic stroke or transient ischemic attack. However, the use of
anticoagulant therapy is also associated with an increased risk of major bleeding. While
the benefit outweighs the risk in most patients, careful consideration of the risk to
benefit ratio is necessary in those at relatively low risk. These issues are discussed
elsewhere.
In addition to atrial fibrillation, other potential cardiac sources of embolism for which
anticoagulation therapy may be indicated for select patients include the following:
Mechanical heart valves and a subpopulation of high-risk patients with
bioprosthetic valves
Left ventricular thrombus
Dilated cardiomyopathy
Rheumatic valve disease
Recent myocardial infarction in high-risk patients
8. 32 yo woman hx 1w progressive sensory loss. Started on feet, felt numbness, slowly
ascending symmetrically to umbilicus. Uninary urgency, frequency, nocturia. Tight band-
like sensation midabdominal. Slowing left eye adduction during saccadic movements.
Diffuse hyperreflexia. Decrease pinprick up to the umbilicus. Dx?
E. Multiple sclerosis
Most patients with MS have relapsing-remitting disease, which typically presents in a
young adult with a clinically isolated syndrome suggestive of MS such as optic neuritis,
long tract symptoms/signs (eg, numbness, paresthesia, or weakness), a brainstem
syndrome (eg, internuclear ophthalmoplegia), or a spinal cord syndrome (eg, transverse
myelitis). The most common clinical presentation of primary progressive MS is a spinal
cord syndrome with spastic paraparesis and no clear sensory level.
Internuclear ophthalmoplegia (INO) refers to abnormal horizontal ocular movements
with lost or delayed adduction and horizontal nystagmus of the abducting eye. It is
caused by a lesion of the medial longitudinal fasciculus in the brainstem on the side of
diminished adduction. Convergence is typically preserved.
Sensory symptoms are the most common initial feature of MS (table 3) and are present
in almost every patient at some time during the course of disease. The sensory features
can reflect spinothalamic, posterior column, or dorsal root entry zone lesions.
Symptoms are commonly described as numbness, tingling, pins-and-needles, tightness,
coldness, or swelling of the limbs or trunk. On testing sensation with a sharp object such
as a pin, patients frequently report that the sharp feeling is increased, or feels like a mild
electric shock, or that the stimulus spreads in a ripple fashion from the point at which it
is applied. A bilateral sensory level is more common than a hemisensory syndrome.
Bowel, bladder, and sexual dysfunction are common in MS. The extent of sphincter and
sexual dysfunction often parallels the degree of motor impairment in the lower
extremities. The most common urinary complaints are frequency and urgency. Urinary
incontinence becomes more common as the disease progresses, and an atonic dilated
bladder that empties by overflow can be the end result.
9. 27 yo woman, hx 1y progressive weakness, numbness, tingling both arms. Strength 3/5
upper extremities, 4/5 lower extremities. Muscle wasting, atrophy forearms and hands
bilat. Deep tendon reflex absent in upper extremities, 4+ in lower extremities. Babinski +
bilat. Decreased pinprick and light touch in upper extremities. Dx?
D. Syringomyelia.
Syringomyelia (/sᵻˌrɪŋɡəmaɪˈiːliə, -ɡoʊ-/[1][2]) is a generic term referring to a disorder in
which a cyst or cavity forms within the spinal cord. This cyst, called a syrinx, can expand
and elongate over time, destroying the spinal cord. The damage may result in pain,
paralysis, weakness,[3] and stiffness in the back, shoulders, and extremities.
Syringomyelia may also cause a loss of the ability to feel extremes of hot or cold,
especially in the hands. The disorder generally leads to a cape-like loss of pain and
temperature sensation along the back and arms. Each patient experiences a different
combination of symptoms. These symptoms typically vary depending on the extent and,
often more critically, to the location of the syrinx within the spinal cord.
10. 72 yo man progressive aching lower extremities during walking for a year. Pain after 15
min of walking, resolve after stop. Progress to 5-10 min after walking, and after standing
of bed. Pain radiates from buttocks to feet. Hx of COPD, smoking, cervical arthritis, neck
and arm pain. Decreased breath sounds bilat. Strenght and sens normal in extremities.
Babinski (-) bilat. Tendon reflex 1+ knees. 2+ upper extremities. Dorsal pedis pulses
normal. Dx?
L. Lumbar spinal stenosis.
Spondylosis, or degenerative arthritis affecting the spine, is the most common cause of
lumbar spinal stenosis and typically affects individuals over the age of 60 years [4].
Obesity may also be a risk factor [5]. Progressive disc degeneration due to aging, trauma,
or other factors can lead to disc protrusion and/or loss of disc height with attendant
loading of the posterior elements of the spine, including the facet joints. Facet joint
arthropathy and osteophyte formation follow, along with hypertrophy of the
ligamentum flavum. All of these processes (facet osteophytes, ligamentum flavum
hypertrophy, and disc bulging) can encroach on the central canal and the neural
foramina.
The erect, extended position narrows the lumbar canal by reducing the interlaminar
space, causing overlap of laminar edges of adjoining vertebral bodies, relaxing and
inward buckling of the ligamentum flavum, and rostral-anterior migration of the superior
facets [3,17]. This may explain the onset or persistence of symptoms with prolonged
standing. It is also possible that increased metabolic demands on spinal nerve roots
during walking may exceed the available microvascular blood flow, especially when
intrathecal pressures are elevated
Neurogenic (or pseudo) claudication is a hallmark of LSS [1]. This is the tendency for
symptoms, usually pain, to be exacerbated with walking, standing, and/or maintaining
certain postures, and relieved with sitting or lying [18]. Many patients with LSS are
symptomatic only when active. In one systematic review of clinical findings in LSS,
neurogenic claudication was moderately sensitive and present in 82 percent of patients
[19]. Exacerbation by prolonged standing in an erect posture was an even more sensitive
but less specific sign. No pain with sitting was the most specific finding. More than half of
patients with LSS also reported relief with sitting or flexion at the waist (squatting,
leaning forward), findings that were relatively specific for LSS (84 and 92 percent
respectively), but less sensitive.
The primary symptoms of LSS include discomfort, sensory loss, and weakness in the legs,
reflecting involvement of spinal nerve roots within the lumbar spinal canal. In the clinical
series described above, pain was the most common symptom, occurring in 93 percent of
patients, followed by numbness and/or tingling in 63 percent, and weakness in 43
percent [18]. Symptoms were bilateral in most (68 percent), but often asymmetrical, and
usually involved the entire leg rather than just the upper or lower leg (78 versus 15 and 6
percent, respectively). Pain in a single nerve root distribution occurred in only 6 percent.
Low back pain occurred in 65 percent and was described as mechanical and mild. Low
back pain in LSS is not necessarily associated with the claudication symptoms.
The neurologic examination is often normal in patients with LSS. The straight leg raising
sign is present only in a minority of patients (10 percent) [18,20]. However, in some
patients with LSS, more prolonged or severe nerve root involvement may lead to fixed
and/or progressive neurologic deficits.
11. 77 yo man hospitalized with antibiotic iv for leg rash is unable to urinate. Rash started 10
days ago from right foot to calf. Admission right lower extremity warm and
erythematous. 1 day ago onset new neck pain. Hx DM2 treated with glyburide. Appears
ill. 102.2F, nuchal rigidity, muscle strength normal upper extremities. Unable to lift lower
extremities against gravity. Tendon reflexes brisk. Babinski (+) bilat. Pinprick decreased
below C7. Dx?
A. Epidural abscess of the spine.
Risk factors for spinal epidural abscesses (SEAs) include epidural catheters, diabetes
mellitus, alcoholism, HIV infection, bacteremia, and intravenous drug abuse.
Approximately one-third of patients with SEA have no identifiable source for the
infection. Among the two-thirds for whom a portal of entry can be identified, the most
common sites of origin are infections of skin and soft tissues and complications of spinal
surgery or other invasive procedures, including epidural catheters that are left in place,
usually for pain control (table 1). Injection drug abuse is also an important predisposing
factor. (See 'Portals of entry' above.)
The initial manifestations of spinal epidural abscess (SEA) are often nonspecific and
include such signs and symptoms as fever and malaise. The classical diagnostic triad
consists of fever, spinal pain, and neurologic deficits. However, only a small proportion
of patients have all three components at presentation [26,30,31]. Fever may be absent in
some patients, leading to delayed or missed diagnosis [26]. In fact, most patients present
to healthcare providers multiple times before the diagnosis is made.
Over time, however, an untreated abscess causes symptoms that progress in a typical
sequence [5]:
●Back pain, which is often focal and severe, then
●Nerve root pain, described as "shooting" or "electric shocks" in the distribution of the
affected nerve root, then
●Motor weakness, sensory changes, and bladder or bowel dysfunction, and then
●Paralysis
Once paralysis develops, it may quickly become irreversible. Thus, urgent intervention
may be required if progression of weakness or other neurologic findings are detected.
12. 57 yo man progressive weakness arms and legs. 6 months ago normal strength. Current,
fasciculation in hands and right lower extremity, and pseudobulbar palsy. Normal sens
and cranial nerves. Oriented and normal affect. MRI brain no abnormalities. Mechanism
of disease?
C. Motor neuron degeneration

pseudobulbar palsy describes impairment of function of cranial nerves IX-XII due to


upper motor neuron lesions of the corticobulbar tracts in the mid-pons. For clinically
evident dysfunction to occur, such lesions must be bilateral as these cranial nerve nuclei
receive bilateral innervation. Pseudobulbar palsy is a medical condition characterized by
the inability to control facial movements (such as chewing and speaking) and caused by a
variety of neurological disorders. Patients experience difficulty chewing and swallowing,
have increased reflexes and spasticity in tongue and the bulbar region, and demonstrate
slurred speech (which is often the initial presentation of the disorder), sometimes also
demonstrating uncontrolled emotional outbursts.[1]
The condition is usually caused by the damage (bilateral degeneration) to the neurons of
the brain stem, specifically to the corticobulbar tract (upper motor neuron tract to
cranial nerve motor nuclei).
Pseudobulbar palsy is the result of damage of motor fibers traveling from the cerebral
cortex to the lower brain stem. This damage might arise in the course of a variety of
neurological conditions that involve demyelination and bilateral corticobulbar lesions.
Examples include:
Vascular causes: bilateral hemisphere infarction, CADASIL syndrome
Progressive supranuclear palsy
Amyotrophic lateral sclerosis
Parkinson's disease and related multiple system atrophy
Various motor neuron diseases, especially those involving demyelination
Multiple sclerosis and other inflammatory disorders
High brain stem tumors
Metabolic causes: osmotic demyelination syndrome[2]
Neurological involvement in Behçet's disease
Brain trauma
ALS has an age distribution that peaks in the seventh to eighth decades. However, ALS
can occur in people in their twenties. The clinical hallmark of amyotrophic lateral
sclerosis (ALS) is the combination of upper motor neuron and lower motor neuron signs
and symptoms. Upper motor neuron findings of weakness, hyperreflexia, and spasticity
result from degeneration of frontal motor neurons. The lower motor neuron findings of
weakness, atrophy or amyotrophy, and fasciculations are a direct consequence of
degeneration of lower motor neurons in the brainstem and spinal cord.
Asymmetric limb weakness is the most common presentation of ALS (80 percent).
Bulbar onset, usually manifested as dysarthria or dysphagia, is the next most common
pattern (20 percent). However, differences in site and segment (cranial, cervical,
thoracic, or lumbosacral) of onset, pattern and speed of spread, and the degree of
upper and lower motor neuron dysfunction produce a disorder that is remarkably
variable between individuals.
Cognitive impairment, typically related to frontotemporal executive dysfunction, may
precede or follow the onset of upper motor neuron and/or lower motor neuron
dysfunction in patients with ALS. Frontotemporal dementia may be associated with ALS
in 15 to 50 percent of cases.
13. 27 yo man ER multiple trauma after motor vehicle collision. initially arousable. Increasing
unresponsive next 45 min. pupil left eye dilated, ptosis left eye. Begins to hyperventilate.
Cause?
K. uncal herniation.
The uncus can squeeze the oculomotor nerve (a.k.a. CN III), which may affect the
parasympathetic input to the eye on the side of the affected nerve, causing the pupil of
the affected eye to dilate and fail to constrict in response to light as it should. Pupillary
dilation often precedes the somatic motor effects of CN III compression called
oculomotor nerve palsy or third nerve palsy. This palsy presents as deviation of the eye
to a "down and out" position due to loss of innervation to all ocular motility muscles
except for the lateral rectus (innervated by abducens nerve (a.k.a. CN VI) and the
superior oblique (innervated by trochlear nerve a.k.a. CN IV). The symptoms occur in this
order because the parasympathetic fibers surround the motor fibers of CN III and are
hence compressed first.
Compression of the ipsilateral posterior cerebral artery will result in ischemia of the
ipsilateral primary visual cortex and contralateral visual field deficits in both eyes
(contralateral homonymous hemianopsia).
Another important finding is a false localizing sign, the so-called Kernohan's notch, which
results from compression of the contralateral[7] cerebral crus containing descending
corticospinal and some corticobulbar tract fibers. This leads to ipsilateral hemiparesis (as
these tracts are above their decussation where they are compressed). Since the
corticospinal tract predominately innervates flexor muscles, extension of the leg may
also be seen.
In central herniation, the diencephalon and parts of the temporal lobes of both of the
cerebral hemispheres are squeezed through a notch in the tentorium cerebelli.[5][8]
Transtentorial herniation can occur when the brain moves either up or down across the
tentorium, called ascending and descending transtentorial herniation respectively;
however descending herniation is much more common.[1] Downward herniation can
stretch branches of the basilar artery (pontine arteries), causing them to tear and bleed,
known as a Duret hemorrhage. The result is usually fatal.[8] Other symptoms of this type
of herniation include small, dilated, fixed pupils with[9] paralysis of upward eye
movement giving the characteristic appearance of "sunset eyes". Also found in these
patients, often as a terminal complication is the development of diabetes insipidus due
to the compression of the pituitary stalk. Radiographically, downward herniation is
characterized by obliteration of the suprasellar cistern from temporal lobe herniation
into the tentorial hiatus with associated compression on the cerebral peduncles.
Upwards herniation, on the other hand, can be radiographically characterized by
obliteration of the quadrigeminal cistern. Intracranial hypotension syndrome has been
known to mimic downwards transtentorial herniation.
14. 67 yo man, hx 6w progressive cognitive changes and twitching movements arms and
legs. Mild ataxia dysarthria. Normal strength, reflexes 3+, ataxic gait, frequent irregular
jerk movements upper and lower extremities. Alert but distractable. Minimental 19/30.
CSF glucose 80, protein 45, WBC (-), RBC(-). MRI vague abnormal T2 signal in basal
ganglia. Additional finding in CSF?
B. Increased 14-3-3 protein concentration.
Rapidly progressive mental deterioration and myoclonus are the two cardinal clinical
manifestations of sCJD. Death usually occurs within one year of symptom onset
Mental deterioration may be manifest as dementia, behavioral abnormalities, and
deficits involving higher cortical function. Concentration, memory, and judgment
difficulties are frequent early signs [67]. Mood changes such as apathy and depression
are common; euphoria, emotional lability, and anxiety occur less frequently. Sleep
disturbances, particularly hypersomnia, but also insomnia, are also common, and may
be a presenting sign [68,69]. With disease progression, dementia becomes dominant in
most patients and can advance rapidly.
Myoclonus, especially provoked by startle, is present in more than 90 percent of
patients at some point during the illness but may be absent at presentation, even when
dementia is profound. sCJD should always be considered in a patient with the
combination of a rapidly progressive dementia and myoclonus.
The Centers for Disease Control and Prevention (CDC) outline the following criteria for
probable sporadic CJD [111]:

Progressive dementia and


At least two out of the following four clinical features: myoclonus; visual or
cerebellar disturbance; pyramidal/extrapyramidal dysfunction; akinetic
mutism and
Atypical electroencephalogram (EEG) during an illness of any duration, and/or a
positive 14-3-3 cerebrospinal fluid (CSF) assay with a clinical duration to death
less than two years, and/or magnetic resonance imaging (MRI) high signal
abnormalities in caudate nucleus and/or putamen on diffusion-weighted
imaging (DWI) or fluid attenuated inversion recovery (FLAIR) and
Routine investigations should not suggest an alternative diagnosis.

Antibodies directed against intracellular neuronal proteins (called classical


paraneoplastic or onconeuronal antibodies) – These antibodies belong to the category
of "well characterized" paraneoplastic antibodies (table 2), and their detection almost
always indicates the presence of an underlying tumor. Examples include Hu (also known
as type 1 anti-neuronal nuclear antibody [ANNA-1]), Ri (also known as type 2 anti-
neuronal nuclear antibody [ANNA-2]), Yo (also known as Purkinje cell cytoplasmic
antibody type 1 [PCA-1]), amphiphysin, Ma2, Tr (also known as delta/notch-like
epidermal growth factor-related receptor [DNER]), collapsin response-mediator protein-
5 (CRMP-5), and recoverin. These antibodies are surrogate markers of the
paraneoplastic disorder, but in most of these disorders, the pathogenic mechanism is
believed to be mediated by cytotoxic T-cells.
Antibodies, paraneoplastic syndromes and associated cancers
Antibody Syndrome Associated cancers
Well characterized paraneoplastic antibodies*
Encephalomyelitis including cortical, limbic, brainstem
Anti-Hu (ANNA-1) encephalitis, cerebellar degeneration, myelitis, sensory SCLC, other
neuronopathy, and/or autonomic dysfunction
Anti-Yo (PCA-1) Cerebellar degeneration Gynecological, breast
Cerebellar degeneration, brainstem encephalitis, opsoclonus- Breast, gynecological,
Anti-Ri (ANNA-2)
myoclonus SCLC
Anti-Tr (DNER) Cerebellar degeneration Hodgkin lymphoma
Encephalomyelitis, cerebellar degeneration, chorea, peripheral
Anti-CV2/CRMP5 SCLC, thymoma, other
neuropathy
Germ-cell tumors of testis,
Anti-Ma proteins¶ Limbic, hypothalamic, brainstem encephalomyelitis
lung cancer, other solid
(Ma1, Ma2) (infrequently cerebellar degeneration)
tumors
Anti-amphiphysin Stiff-person syndrome, encephalomyelitis Breast, lung cancer
Anti-recoverinΔ Cancer-associated retinopathy SCLC
Anti-bipolar cells of
Melanoma-associated retinopathy Melanoma
the retina◊
Partially-characterized paraneoplastic antibodies*
Anti-Zic 4 Cerebellar degeneration SCLC
Anti-ANNA-3 Sensory neuronopathy, encephalomyelitis SCLC
Anti-PCA2 Encephalomyelitis, cerebellar degeneration SCLC
Antibodies that occur with and without cancer association
Anti-VGCC Lambert-Eaton myasthenic syndrome, cerebellar dysfunction SCLC
Anti-AChR Myasthenia gravis Thymoma
Multistage syndrome with memory and behavioral
Anti-NMDAR disturbances, psychosis, seizures, dyskinesias, and autonomic Teratoma
dysfunction
Anti-AMPAR Limbic encephalitis, psychiatric disturbances Variable solid tumors
Anti-GABA(A)
Encephalopathy, refractory seizures Thymoma
receptor
Anti-GABA(B)
Seizures, limbic encephalitis SCLC
receptor
Anti-LGI1 (previously
Limbic encephalitis, seizures Thymoma, SCLC
attributed to VGKC)
Anti-Caspr2
Thymoma and variable solid
(previously attributed Morvan syndrome and some patients with neuromyotonia
tumors
to VGKC)
Anti-nAChR Subacute pandysautonomia SCLC, others
Encephalomyelitis with muscle spasms, rigidity, myoclonus, Often without cancer, one
Anti-GlyR
hyperekplexia patient with lung cancer
Hodgkin lymphoma§ or no
Anti-mGluR5 Limbic encephalitis, prosopagnosia, involuntary movements
tumor
Hodgkin lymphoma or no
Anti-mGluR1 Cerebellar degeneration
tumor

15. 4 months girl, ER b/c drowsiness 3 day hx. Also, feeding poorly, spitting up after feeding.
Uncomplicated pregnancy and delivery. Arrival, responsive to tactile stimuli. 25 th
percentile for length, 90th percentile for head circumference. 98F, resp 24/min, BP
100/70. Bulging anterior fontanel. Hyperpigmented macule with hair growth in upper
lumbar spine noted at birth. Dx?
B. Chiari type 2 malformation

A sagittal T1-weighted MRI in a pediatric patient shows several characteristic intracranial


findings of the Chiari II malformation, including downward displacement of cerebellar
tissue through the foramen magnum (arrow), a small fourth ventricle (arrowhead), and
tectal beaking (dashed arrow).

Chiari I malformation (CM-I) is characterized by abnormally shaped cerebellar tonsils that


are displaced below the level of the foramen magnum.

Chiari II malformation (CM-II), also known as Arnold-Chiari malformation, is


characterized by downward displacement of the cerebellar vermis and tonsils, a
brainstem malformation with beaked midbrain on neuroimaging, and a spinal
myelomeningocele.
The Chiari II malformation (CM-II) is characterized by downward displacement of inferior
cerebellar vermis (involving the nodulus, pyramis and uvula), and cerebellar tonsils and
medulla through the foramen magnum into the upper cervical canal, in association with
a myelomeningocele at the lumbosacral or occasionally a higher level of the spinal cord
(image 2). The malformation obstructs the outflow of cerebrospinal fluid through the
posterior fossa, causing hydrocephalus. Almost all patients with a myelomeningocele
have CM-II, and most have associated hydrocephalus. A reduced volume of the posterior
fossa with an enlarged foramen magnum and low torcula with ventral displacement of
the tentorium cerebelli are constant features.
16. 77 yo woman, 3 day hx visual difficulties. Off-balance. Denting car while parking. Hx 2
year AF treated with aspirin. 98F, pulse 95/min, resp 22/min, BP 110/80. Left homonimus
hemianopia. Strength and sens normal. Speech normal. Alert and oriented. Read only
words in right side page. Infarct what territory cerebral artery?
F. Right posterior

Occipital lobe lesions are the most common cause of homonymous hemianopia and are
most often vascular in origin [8-10]. Lesions of the occipital lobe tend to produce an
isolated homonymous hemianopia (ie, not associated with other neurologic deficits).
Depending on the location of the lesion, various visual field patterns are seen.
18. 57 yo man progressive weakness both legs for 5 months. Problem arising from chair and
climbing stairs. Persistent cough, weigh loss, dry mouth. Smoking for 40y. weakness hip
flex and knee extension. No muscle tenderness. Reflexes absent. Babinski (-). Normal
sens. ESR 95, CK activity 25U/L. CXR right upper lobe mass. Localization abnormality?
C. Myoneural junction
The clinical features of LEMS were well defined by Lambert and Eaton in Minnesota [29]
and by O'Neill and colleagues in England [18]. Most patients present with complaints of
slowly progressive proximal muscle weakness, and this feature is present in almost all
patients at some point in the illness. Occasional patients have a subacute or acute
presentation.
Patients typically describe an alteration in gait or difficulty arising from a chair or
managing stairs. Many describe aching or stiff muscles. Muscle fatigability or cramping is
common, particularly after protracted exercise. There tends to be relative preservation
of distal muscle function.
Symmetrical muscle involvement is the rule. Upper extremity complaints are usually
modest and typically involve proximal muscle function. Patients with LEMS and small
cell lung cancer typically develop a more rapidly progressive course of proximal and
then distal arm muscle weakness than patients who have non-tumor LEMS [30].
Autonomic dysfunction is often present and can be an important clue to the diagnosis.
Dry mouth from reduced salivation is the most common autonomic symptom and
occasionally is the presenting complaint, while erectile dysfunction is common in men
with LEMS. In the largest study of autonomic dysfunction in LEMS involving 30 patients,
dry mouth was identified in 77 percent, and impotence was present in 45 percent of all
men [31]. Other symptoms may include blurred vision and constipation.
Cranial nerve involvement is present in a minority but is typically less severe and less
striking than that seen in MG. Ocular symptoms, particularly ptosis, are the most
common cranial nerve manifestation with LEMS. Oropharyngeal symptoms of dysarthria,
dysphagia, and difficulty chewing have also been occasionally observed.
Antibodies directed against the voltage-gated calcium channel (VGCC), a large
transmembrane protein with multiple subunits, play a central role in the
pathophysiology of LEMS. These antibodies interfere with the normal calcium flux
required for the release of acetylcholine. P/Q-type VGCCs make up more than 95
percent of the functioning receptors at the neuromuscular junction (NMJ) and probably
represent the main immunologic target in LEMS.
19. 67 yo woman confused and belligerent one day after cholecystectomy. Hx 2y mild
dementia, Alzheimer’s. no medication. IM haloperidol administered. 1h later present
hyperextended neck. Repetitive conjugate upward movement of eyes. Most appropriate
treatment?
A. Anticholinergic
Acute dystonias are involuntary contractions of major muscle groups, and are
characterized by symptoms such as torticollis, retrocollis, oculogyric crisis, and
opisthotonos. These are usually rapid in onset and highly disturbing to patients. An
extremely rare dystonia, laryngospasm, can be life threatening. Risk factors for dystonia
include young age, male sex, use of cocaine, and a history of acute dystonic reaction.
Dystonias that are very disturbing can be treated with 1 to 2 mg of benztropine or 50 mg
of diphenhydramine daily, administered intravenously or intramuscularly. Milder
dystonias can be treated with oral benztropine 1 to 2 mg once or twice daily. (See
"Classification and evaluation of dystonia".)
The emergence of a dystonia should lead to a reevaluation of the patient's antipsychotic
regimen. After acute treatment of the dystonia, the preferred intervention is changing
to an antipsychotic with a lower liability for EPS (table 1). If changing the antipsychotic
is not an option, adding an anticholinergic antiparkinson medication such as
benztropine 1 to 2 mg by mouth twice daily may be effective.
Prophylactic treatment with an anticholinergic agent (benztropine or diphenhydramine)
is recommended to prevent an acute dystonic reaction in patients who receive
intramuscular haloperidol (eg, in the treatment of acute agitation or psychosis).
Prophylactic treatment is particularly important in individuals with little prior exposure
to antipsychotics. As an example, intramuscular haloperidol 5 or 10 mg can be
accompanied by intramuscular benztropine 1 or 2 mg.
20. 72 yo woman, 2h sudden onset dim vision right eye. Symptoms started 3w ago
generalized stiffness, aches morning, bitemporal headache. Firm tender temporal
arteries bilat. 20/200 right eye, 20/40 left eye. Hct 32%, WBC 6000, Platelet 300.000, ESR
110. Initial step management?
C. Corticosteroid therapy now and temporal artery biopsy within 3 day
The diagnosis of giant cell arteritis (GCA) should be considered in a patient over the age
of 50 who complains of or is found to have:
New headaches
Abrupt onset of visual disturbances
Symptoms of polymyalgia rheumatica
Jaw claudication
Unexplained fever or anemia
High erythrocyte sedimentation rate (ESR) and/or high serum C-reactive protein
(CRP)
Glucocorticoid treatment should be instituted promptly once the diagnosis of GCA is
suspected strongly, often even before it is confirmed. For patients in whom the
diagnostic suspicion of GCA is high, especially those with recent or threatened vascular
complications such as visual loss, therapy should be started immediately. A temporal
artery biopsy should be obtained as soon as possible, but treatment should not be
withheld while awaiting the performance or the results of the biopsy.
If there is a strong suspicion of GCA as the cause of visual loss, we suggest use of
intravenous pulse methylprednisolone. In this setting, the typical dose is 1000 mg
intravenously each day for three days. This is then followed by oral therapy with 1
mg/kg per day (maximum of 60 mg/day), as recommended above for uncomplicated
GCA.
21. 47 yo man hx 8 month progressive severe bifrontal headache. Scant beard, sallow waxy
complexion. Bilat temporal hemianopia. Visual acuity normal. Dx?
G. Pituitary adenoma.
Impaired vision is the most common symptom that leads a patient with a nonfunctioning
adenoma, of which over 80 percent are gonadotroph adenomas, to seek medical
attention. Visual impairment is caused by suprasellar extension of the adenoma, leading
to compression of the optic chiasm. The most common complaint is diminished vision in
the temporal fields (bitemporal hemianopsia). One or both eyes may be affected and, if
both, to variable degrees. Diminished visual acuity occurs when the optic chiasm is more
severely compressed. Other patterns of visual loss can also occur. Thus, an intrasellar
lesion should be suspected when there is any unexplained pattern of visual loss.
Headaches, presumably caused by expansion of the sella. The quality of the headache is
not specific.
Other neurologic symptoms that may cause a patient with a sellar mass to seek medical
attention include:
●Diplopia, induced by oculomotor nerve compression resulting from lateral
extension of the adenoma.
●Pituitary apoplexy induced by sudden hemorrhage into the adenoma, causing
excruciating headache and diplopia.
●Cerebrospinal fluid rhinorrhea, caused by inferior extension of the adenoma,
an extremely uncommon presentation.
●Parinaud syndrome, a constellation of neuroophthalmologic findings, most
often paralysis of upward conjugate gaze, that result from ectopic pinealomas.
At the time of initial presentation with a neurologic symptom, many patients with sellar
masses, when carefully questioned, admit to symptoms of pituitary hormone
deficiencies. However, these symptoms are not usually the reason that the patient seeks
medical attention. The most common pituitary hormone deficiencies are of
gonadotropins, resulting in hypogonadism in both men and women
Craniopharyngiomas are solid or mixed solid-cystic benign tumors that arise from
remnants of Rathke's pouch along a line from the nasopharynx to the diencephalon.
Most are either intrasellar or suprasellar. About 50 percent present clinically during
childhood and adolescence, the other 50 percent present after age 20 years, some not
until age 70 or 80 years. The major presenting symptoms are growth retardation in
children and abnormal vision in adults. In addition, pituitary hormonal deficiencies,
including diabetes insipidus, are common. Direct damage to or compression of normal
structures can lead to a range of endocrine abnormalities. Frequently observed
complications include deficiencies of growth hormone, gonadotropin, thyroid stimulating
hormone, and adrenocorticotropic hormone in an estimated 75, 40, 25 and 25 percent of
cases, respectively [4]. Diabetes insipidus is frequent when the pituitary stalk is involved.
Growth failure, which can be caused by either hypothyroidism or growth hormone
deficiency, is the most common presentation in children. Sexual dysfunction is the most
common endocrine manifestation in adults; almost 90 percent of men complain of
erectile dysfunction, while most women have amenorrhea.
22. 52 yo 6-w financial advisor hx progressive right hand weakness, interfere with writing
ability. BMI 28. Decreased active abduction small and index fingers. No crossing right
index and middle fingers. Unable to pinch the plmar apect of the right thumb agains the
radial side of the index finger without flexing the interphalangeal joint of the thumb to
provide strength. . Flexion interphalangeal joint right hand strength 5/5. Sens normal.
Next step for dx?
c. Electrophysiological studies.
Carpal tunnel syndrome, The hallmark of classic CTS is pain or paresthesia (numbness
and tingling) in a distribution that includes the median nerve territory, with involvement
of the first three digits and the radial half of the fourth digit (figure 1). The symptoms of
CTS are typically worse at night and often awaken patients from sleep. Some patients
react to these symptoms by shaking or wringing their hands or by placing them under
warm running water [1].
Although the sensory symptoms of CTS are usually limited to the median-innervated
fingers, there can be significant variability. The pain and paresthesia may be localized to
the wrist or involve the entire hand. It is not uncommon for sensory symptoms to
radiate proximally into the forearm, and less frequently to radiate above the elbow to
the shoulder, but the neck is not affected.
Fixed sensory loss is usually a late finding characterized by a distinctive clinical pattern
that involves the median-innervated fingers and spares the thenar eminence. This
pattern occurs because the palmar sensory cutaneous nerve arises proximal to the wrist
and passes over, rather than through, the carpal tunnel.
In more severe cases of CTS, motor involvement leads to complaints of weakness or
clumsiness when using the hands, such as difficulty holding objects, turning keys or
doorknobs, buttoning clothing, or opening jar lids [1]. Clinical signs may include
weakness of thumb abduction and opposition, and atrophy of the thenar eminence.
CTS is a clinical diagnosis. The diagnosis is suspected when the characteristic symptoms
and signs are present. The most important of these are nocturnal pain or paresthesia in
the distribution of the median nerve.
Electrodiagnostic testing can be helpful to confirm or exclude CTS when the clinical
diagnosis is uncertain [5]. It is also useful to gauge severity of nerve compression and to
aid in decisions regarding surgical intervention.
Electrodiagnostic testing, primarily with nerve conduction studies (NCS), sometimes
supplemented with needle electromyography (EMG), is a standard part of the
evaluation for CTS because it has a high sensitivity and specificity for confirming the
diagnosis.
23. 47yo man with 2 week progressive weakness. Feet flop while walking. Trips walking
upstairs. Difficultystanding up from low chairs and opening jars. Constipation but no
changes in bladder/sexual function. Smoke. Alcohol. 3-4 beers/day, 6 weeks drinking
home-distilled whiskey. Atrophy of intrinsic muscles hand and feet. Strength 4/5. Deep
tendon reflexes 1+. Babinski (-). Sens normal. Microcytic anemia and hyperuricemia.
Cause?
E. Lead poisoning
Early symptoms of lead poisoning in adults are commonly nonspecific and include
depression, loss of appetite, intermittent abdominal pain, nausea, diarrhea,
constipation, and muscle pain.[21] Other early signs in adults include malaise, fatigue,
decreased libido, and problems with sleep.[15] An unusual taste in the mouth and
personality changes are also early signs.[22] .
in adults, symptoms can occur at levels above 40 μg/dL, but are more likely to occur
only above 50–60 μg/dL.
At blood lead levels between 25 and 60 μg/dL, neuropsychiatric effects such as delayed
reaction times, irritability, and difficulty concentrating, as well as slowed motor nerve
conduction and headache can occur.[25] Anemia may appear at blood lead levels higher
than 50 μg/dL.
In adults, abdominal colic, involving paroxysms of pain, may appear at blood lead levels
greater than 80 μg/dL.[16] Signs that occur in adults at blood lead levels exceeding
100 μg/dL include wrist drop and foot drop, and signs of encephalopathy (a condition
characterized by brain swelling), such as those that accompany increased pressure
within the skull, delirium, coma, seizures, and headache.
A peripheral neuropathy that frequently manifests with extensor weakness or
"wrist/ankle drop" due to an axonal degeneration that primarily affects motor nerves
[54]. This is seen less frequently since governmental regulations have resulted in lower
lead exposures at work. In rare situations of high-level lead poisoning, tremor has also
been reported.
Lead affects the peripheral nervous system (especially motor nerves) and the central
nervous system.[18] Peripheral nervous system effects are more prominent in adults
and central nervous system effects are more prominent in children.[24] Lead causes the
axons of nerve cells to degenerate and lose their myelin coats.[18]
24. 32 yo woman ER progressive visual dimming right eye since awakening 8h ago.
Symptoms started with a mild headache, over right eye before going to bed. Headache
is different than previous usual migraines treated with propranolol. cannot read, varely
count fingers in right eye. CT normal. Dx?
Maybe E. Optic neuritis
Optic neuritis is an inflammatory, demyelinating condition that causes acute, usually
monocular, visual loss. It is highly associated with multiple sclerosis (MS). Optic neuritis
is the presenting feature of MS in 15 to 20 percent of patients and occurs in 50 percent
at some time during the course of their illness.
Clinical features of more common optic neuropathies*
Non-arteritic
Arteritic ischemic Leber's hereditary
Optic neuritis ischemic optic Neuroretinitis
optic neuropathy optic neuropathy
neuropathy
Age 20 to 50 years >50 years >70 years 25 to 40 years Children
80 to 90 percent
Gender 2:1 female Equal 3.5:1 female Equal
male
Headache, scalp
Present in <10
Pain Present in >90 percent tenderness, jaw Not present Variable
percent
claudication
Onset Hours to days Sudden Sudden Weeks to months Hours to days
Usually
unilateral; low Bilateral - but
Unilateral or May occur in both
Usually unilateral chance may presentation often Often bilateral
bilateral eyes in rapid sequence
recur in other unilateral
eye years later
Pale swelling of disc;
fundus may also be Disc hyperemia
Papillitis,
Funduscopic Papillitis present in Papillitis present normal (posterior but no swelling;
macular edema,
examination one-third in most ischemic optic peripapillary
exudates
neuropathy - indicates telangiectasia
giant cell arteritis)
Altitudinal Altitudinal or
Visual field Central or
Central scotoma (usually generalized Variable
defect cecocentral defect
inferior) defect constriction
Inflammation of optic
Magnetic nerve in most (one-
resonance third to one-half will May show Variably
Often normal Normal
imaging: optic have other enhancement abnormal
nerve demyelinating
lesions)
Over several
Begins within two to Poor once vision loss
months, only 40 One-third achieve
four weeks, most has occurred; may Most recover
Prognosis percent improve some
achieve 20/40 or cause rapid blindness fully
by three or more improvement
better untreated
lines

differential dx
Not C.Retinal migraine is a rare condition that is characterized by repeated
attacks of monocular scotomata or blindness lasting less than one hour,
associated with or followed by headache. The International Headache Society
prefers the term retinal migraine [76], but ocular migraine has been suggested as
a more precise term, since both retinal and ciliary circulations may be involved
[97]. Occasionally the onset may be abrupt and difficult to distinguish from
amaurosis fugax [79]. (See "Amaurosis fugax (transient monocular or binocular
visual loss)".)
Irreversible visual loss may be a complication of retinal migraine, although the
incidence is uncertain. In one of the largest studies to date that reported 6 new
cases and reviewed 40 from the literature, permanent visual loss was eventually
present in 20 patients (43 percent) [98]. No predictors of irreversible visual loss
could be identified, and no consistent pattern of visual loss was observed among
these patients. However, permanent visual loss may be less frequent than
suggested by these data, since it is likely that cases with such a major
complication are more apt to be identified and to be reported (ie, reporting bias).
25. 10 yo after Thankgiving, with hx 4-day rapid irregular involuntary movements of her
face, trunk, extremities. During episodes, she has been very anxious irritable, and
headaches. She has been worrying obsessively. No previous serious illness, except
migratory joint pain. Examination, restless appear, generalized involuntary movements
of face trunk , and extremities. Cause?
A. Autoinmune disorder of the basal ganglia
Pediatric autoimmune neuropsychiatric disorder associated with group A streptococci
(PANDAS) is a term used to describe a subset of children whose symptoms of obsessive
compulsive disorder (OCD) or tic disorders are exacerbated by group A streptococcal
(GAS) infection [1]. The hypothesized association between PANDAS and GAS is
controversial, as is the limitation of the diagnosis exclusively within the pediatric age
group
Most of the knowledge about PANDAS has been obtained by studying patients with a
known tic disorder or long-standing obsessive-compulsive disorder (OCD) in research
facilities and referral centers [1-4]. Investigators at these centers noted an association
between Sydenham chorea and OCD [5-7]. Sydenham chorea is a movement disorder
characterized by chorea, emotional lability, and hypotonia. It is one of the major clinical
manifestations of acute rheumatic fever (ARF). The investigators also identified a subset
of children with OCD or tic disorders following group A streptococcal (GAS) infection
who did not meet criteria for Sydenham chorea [4,8,9]. This subset is described by the
acronym PANDAS [1].
The diagnostic criteria for PANDAS, which are discussed in greater detail below, include
the following [1,10]:
●OCD and/or tic disorder (Tourette disorder, chronic motor or vocal tic disorder)
●Pediatric onset (between three years and onset of puberty)
●Abrupt onset and episodic course of symptoms
●Temporal relation between GAS infection and onset and/or exacerbation
●Neurologic abnormalities, such as motoric hyperactivity, choreiform
movements, or tics during exacerbations
The proposed model for pathogenesis of PANDAS suggests that group A streptococcal
(GAS) infection in a susceptible host causes an abnormal immune response with
resultant central nervous system manifestations. If this hypothesis is correct, there
might be a role for prophylactic antibiotics and/or immune modulating therapies in the
prevention and treatment of PANDAS. However, the hypothesis is as yet unproven.

Facioscapulohumeral muscular dystrophy (FSHMD, FSHD or FSH)—originally named


Landouzy-Dejerine[2]—is a usually autosomal dominant inherited form of muscular
dystrophy (MD)[3] that initially affects the skeletal muscles of the face (facio), scapula
(scapulo) and upper arms (humeral). FSHD is the third most common genetic disease of
skeletal muscle. A progressive skeletal muscle weakness usually develops in other areas
of the body as well; often the weakness is asymmetrical. Non-muscular symptoms
frequently associated with FSHD include subclinical sensorineural hearing loss and
retinal telangiectasia. In more than 95% of known cases, the disease is associated with
contraction of the D4Z4 repeat in the 4q35 subtelomeric region of Chromosome 4.

Conversion disorder, Diagnostic criteria were established for adults predominantly, and
these criteria generally are extended to children. There are no separate child-specific
criteria. Generally, diagnoses in children and adolescents are more difficult because the
expression of emotional distress in the form of physical complaints is developmentally
appropriate in younger children. Younger children lack the capacity to accurately
verbally report symptoms and emotional distress. However, when physical symptoms
are persistent and a child's functioning deteriorates, consideration of a somatic
symptom group of disorders can be considered.
26. 57 yo woman, Hx 3-month gradually progressive weakness shoulder and hip.
Hypothyroidism treated with thyroxine. Normal vital signs. Moderate bilat weakness
deltoid, biceps. Triceps, psoas, glutei, cuadriceps. ESR 85, CK 2456. Normal urianalysis
and blood count. Next step in Dx?
B. Electromyography and nerve conduction studies.
Muscle weakness is the most common feature of DM and PM; over 90 percent of
patients with PM present with muscle weakness [9]. However, cutaneous
manifestations often precede or accompany weakness, which is found at presentation in
only 50 to 60 percent of patients with DM [9,10]. Typically mild myalgias and muscle
tenderness occur in 25 to 50 percent of cases. (See 'Skin findings' below.)
The distribution of weakness is characteristically symmetric and proximal in both PM
and DM. Affected muscles typically include the deltoids and the hip flexors. Weakness of
the neck flexors is also common. Distal muscle weakness, if present, tends to be mild
and usually does not cause significant functional impairment. Rarely, patients present
with focal myositis that usually but not always progresses to the typical generalized
form over time [14].
Patients usually report a history of the insidious or subacute development of the muscle
weakness, with gradual worsening over a period of several months before medical
attention is sought. However, an acute onset of weakness is occasionally reported.
Patients may describe increasing difficulty climbing stairs, getting up from a chair,
carrying heavy groceries, or picking up their children due to the proximal muscle
involvement. They may notice joint pain and swelling, if present, and they occasionally
mistakenly ascribe weakness to the joint involvement. Pain is mild, if present, and
stiffness is not a prominent complaint.
Several laboratory findings are characteristic of dermatomyositis (DM) and polymyositis
(PM). These include:
●Elevated levels of muscle enzymes [9,54]
●Autoantibodies, including antinuclear antibodies, in up to 80 percent of
patients with DM and PM [9,10,54,55]; myositis-specific autoantibodies, in at
least 30 to 40 percent of patients [37,56-58]; and myositis-associated
autoantibodies, especially in patients with overlap syndromes
●Elevated levels of serum and urine myoglobin *59,60+
The erythrocyte sedimentation rate (ESR) is often normal or is only mildly
elevated, even in patients with active muscle disease [5].
The level of serum CK can vary widely. In untreated patients with active muscle disease,
it is usually more than 10-fold the upper limit of normal (ie, at least 2000 to 3000
international units/L). In severe cases, the serum CK concentration may be elevated
more than 50-fold or even 100-fold (ie, up to 10,000 to 20,000 international units/L),
and higher levels are sometimes seen.
in patients in whom the initial evaluation is consistent with possible DM or PM, further
studies should include testing for antinuclear and myositis-specific antibodies (MSA) and
a chest radiograph. Additional diagnostic studies, including muscle or skin biopsy,
electromyography (EMG), and magnetic resonance imaging (MRI) of skeletal muscle,
depend upon the clinical presentation.
EMG shows evidence of muscle membrane irritability, usually including increased
insertional activity, spontaneous fibrillations, positive sharp waves, and complex
repetitive discharges. EMG findings are helpful in confirming the presence of a
myopathic process and in indicating which muscle groups are most involved. This
information can be valuable in selecting a site for muscle biopsy. However, EMG findings
are not specific for either DM or PM, and the EMG is normal in approximately 10
percent of patients.
We diagnose DM without a biopsy in patients with symmetrical proximal muscle
weakness, elevated muscle enzymes, a rash that is relatively specific for DM (such as
Gottron’s papules or a heliotrope rash), and the absence of another explanation for
the findings, as well as in patients with findings of a typical rash, myositis-specific
antibodies, and other features characteristic of the antisynthetase syndrome.
27. 72 yo man 7-day hx increasing severe pain around waist. feels like a burning belt from
the left midback to the umbilicus. Hx of chroniclow back pain, DM2 treated with
glyburide, asthma treated with beclomethasone and albuterol inhalers. Treated with 10-
day corticosteroids for asthma 3 weeks ago. Numbness in T-12 distibution. Most likely
explanation?
C. Diabetic radiculopathy
Upper limb and thoracic involvement has also been observed as part of the syndrome of
diabetic amyotrophy [6,10]. Some have pain or weakness involving the chest or
abdominal wall, suggesting a thoracic radiculopathy, while others have symptoms
consistent with a brachial plexopathy [6,10]. Arm involvement occurs in up to one-third
of patients and may be in the form of mononeuropathies of the ulnar and median
nerves, or may affect more proximal sites in the brachial plexus [10,11]. Most upper
limb symptoms occur in association with lumbosacral plexus involvement.
In one retrospective series of 85 patients diagnosed with diabetic cervical radiculoplexus
neuropathy who presented with pain, paraesthesia or weakness involving the cervical
nerve roots, brachial plexus, or upper extremity nerves, the following observations were
made
●The median age was 62 years (range 32 to 83 years).
●The most common symptom at onset was pain in 62 percent. The most
frequent symptoms at evaluation were weakness, pain, and numbness in 99, 81,
and 66 percent, respectively. Onset was unilateral in 81 percent, with eventual
involvement of the contralateral side in 35 percent. Autonomic symptoms, such
as orthostatic dizziness and changes in sweating, were noted in 15 percent.
●Involvement of the upper, middle, and lower brachial plexus was
approximately equal, with each occurring in 50 to 60 percent, and pan
plexopathy was present in 30 percent. Involvement of lumbosacral and thoracic
regions was noted in 24 and 19 percent. Some patients had isolated or
superimposed involvement of the phrenic, long thoracic, axillary, suprascapular,
or anterior interosseus nerves.
●Type 2 and type 1 diabetes mellitus were present in 79 and 7 present. Most of
the remaining patients had impaired fasting glucose, gestational diabetes, or
steroid-induced diabetes.
●Among 28 patients who had lumbar puncture, the most common cerebrospinal
fluid abnormality was elevated protein.
●Neurophysiologic testing showed a predominantly axonal neuropathy; nerve
biopsy, obtained in 11 patients, was interpreted as showing mainly ischemic
injury from microvasculitis.
●Weight loss >10 lbs was recorded in 35 percent.

28. 32 yo woman breast CA metastasis to bone, brought to ER 12-hour hx severe pelvic pain
and numbness. Medications hydrocodone. 99F, pulse 104/min, resp 18/min, BP 152/70.
Decreased rectal sphincter tone. Strength 4/5 lower extremities. Decreased sens light
touch in saddle distribution. Suprapubic fullness. Catheter yields to 800 mL of urine,
resolves pain but no numbeness. MRI shows tumor on L4-S1 intrathecal. Next step
management?
B. IV administration of high dose Dexamethasone
Management of patients with Neoplastic epidural spinal cord compression (ESCC)
includes the immediate administration of glucocorticoids in nearly all patients, followed
by surgery, external beam radiation therapy (EBRT), or stereotactic body radiotherapy
(SBRT). Systemic therapy may be beneficial in patients with chemosensitive tumors.
High-dose corticosteroid therapy is generally considered to be part of the standard
regimen for ESCC, despite limited documented evidence of benefit and a significant risk
of serious side effects [1,16,17]. Several studies have suggested that lower doses can be
effective but they have not been assessed in randomized trials.
For patients with ESCC and either neurologic symptoms or substantial thecal sac
compression by imaging, we recommend glucocorticoids as an integral component of
the initial management (Grade 1B). (See 'Glucocorticoids' above.)
●For patients with severe neurologic deficits (eg, paraparesis or paraplegia), we
suggest high-dose glucocorticoid therapy (eg, dexamethasone 96 mg
intravenously followed by 24 mg four times daily for three days and then
tapered over 10 days) (Grade 2C).
●For patients with minimal neurologic symptoms, we suggest moderate doses of
glucocorticoids (eg, dexamethasone as a bolus of 10 mg intravenously followed
by 16 mg daily orally in divided doses) (Grade 2C).
●We suggest that glucocorticoids be omitted in patients with small epidural
lesions and a normal neurologic examination
29. 25 yo man, hx 4-month increasing low back most severe on awakening in the morning.
Pain relieved by stretching. No trauma Hx. Occasional dysuria. Decreased forward
flexion of the spine and tenderness over the lumbosacral area and at the insertion site
of the achiles tendon. Dx?
B. Ankylosing spondylitis
Ankylosing spondylitis (AS), a form of spondyloarthritis (SpA), is a chronic inflammatory
disease of the axial skeleton manifested by back pain and progressive stiffness of the
spine; it can also involve the hips, shoulders, peripheral joints, entheses, and digits.
Extraarticular manifestations, including uveitis, may also be seen in patients with AS and
other forms of SpA. AS typically develops in young adults, with a peak age of onset
between 20 and 30 years. The endpoint, which occurs in only a subset, is complete
spinal fusion with hyperkyphosis.
Almost all patients with AS report back pain, which frequently, but not invariably, has
characteristics suggesting an inflammatory etiology. Inflammatory back pain typically
exhibits at least four of the following five features:
●Age of onset <40 years
●Insidious onset
●Improvement with exercise
●No improvement with rest
●Pain at night (with improvement upon arising)
The enthesis is the region of attachment of tendons and ligaments to bone; enthesitis,
inflammation of the enthesis, is a classic feature of AS and other spondyloarthritides.
Enthesitis manifests as pain, stiffness and tenderness of insertions, usually without
much swelling, although swelling is a prominent feature at the Achilles tendon
attachment.
30. 13 yo girl headaches accompanied by nausea and vomiting over past 2 weeks. Located
behind left eye and improve after sleep. Headaches started during final examinations,
missing 2-3 days at school. 98.6F, pulse 90/min, resp 20/min, BP 95/60. Dx?
C. Migraine
Migraine is the most common acute and recurrent headache syndrome in children. It is
characterized by periodic episodes of paroxysmal headache accompanied by nausea,
vomiting, abdominal pain, and relief with sleep.
The onset of migraine tends to be earlier in boys than girls (7 versus 10 years of age) [3].
Thus, before age seven, affected boys outnumber girls. The male preponderance
persists until puberty; by 10 to 14 years of age, more females are affected.
The headache of migraine tends to have a throbbing or pulsatile quality, especially as
the intensity increases. However, the symptoms of migraine vary with age. In toddlers,
who generally are unable to voice complaints about headache, symptoms appreciated
by caregivers include episodic pallor, unusual decreased activity, and vomiting [2,45]. In
children, the headache is more often bifrontal, bitemporal, or generalized than
unilateral [42]; bifrontal headache is more common in younger children, while
bitemporal headache is more common in the early teenage years [46]. The migraine
attack is often accompanied by nausea and sensitivity to light and noise. Many children
have vomiting that occurs once or repetitively.
Characteristics of common headache syndromes in children and adolescents
Tension-type Trigeminal autonomic cephalagia (eg,
Symptom Migraine
headache cluster headache)
Commonly bilateral in young children;
in adolescents and young adults, Always unilateral, usually begins around
Location Bilateral
unilateral in 60 to 70% and bifrontal or the eye or temple
global in 30%
Pressure or
Gradual in onset, crescendo pattern; Pain begins quickly, reaches a crescendo
tightness that
Characteristics pulsating; moderate or severe intensity; within minutes; pain is deep, continuous,
waxes and
aggravated by routine physical activity excruciating, and explosive in quality
wanes
Patient may
Patient Patient prefers to rest in a dark, quiet
remain active or Patient remains active
appearance room
may need to rest
Duration 2 to 72 hours Variable 30 minutes to 3 hours
Ipsilateral lacrimation and redness of the
Nausea, vomiting, photophobia*,
eye; stuffy nose; rhinorrhea; pallor;
Associated phonophobia*; may have aura (usually
None sweating; Horner syndrome; focal
symptoms visual, but can involve other senses or
neurologic symptoms rare; sensitivity to
cause speech or motor deficits)
alcohol
The most common primary headaches in children are migraine and tension-type
headache (table 1). Trigeminal autonomic cephalalgias (including cluster headaches) are
rare in children younger than 10 years.
Tension-type headaches are characterized by headaches that are diffuse in location,
non-throbbing, mild to moderate severity, and do not worsen with activity (although the
child may not wish to participate in activity), and can last from 30 minutes to 7 days
(table 2B). Tension-type headache may be associated with nausea, photophobia or
phonophobia but usually is not accompanied by vomiting [1]. The overlap of some of
these symptoms with those of migraine can make differentiation between the two
headache types difficult [24]. The International Classification of Headache Disorders, 3rd
edition (beta version) (ICHD-3b) specifies that migraine diagnosis takes priority over the
diagnosis of tension-type headache, so when in doubt between the two, the diagnosis
of migraine should be made [1]. Tension-type headaches in children are discussed
separately.
31. 52 yo man HIV (+), 1-month Hx of forgetfulness and inapropiate behavior. Buying
numerous items on internet. Diarrhea. Noncompliant ttmt with antiretrovirals. Weight-
loss, BMI 15. 101.3F, resp 16/min, BP 110/60. Thrush. Labile affect. Recall 0/5 objects
after 5 min. CT shows atrophy cortex and demyelination in subcortical white matter.
Dx?
E. Progressive multifiocal leukoencephalopathy.
Progressive multifocal leukoencephalopathy (PML) is a severe demyelinating disease of
the central nervous system that is caused by reactivation of the polyomavirus JC (JC
virus) [1-3]. Asymptomatic primary infection with JC virus occurs in childhood and
antibodies can be found in 86 percent of adults [4]. In most individuals, JC virus remains
latent in kidneys and lymphoid organs, but, in the context of profound cellular
immunosuppression, JC virus can reactivate, spread to the brain, and induce a lytic
infection of oligodendrocytes, which are the CNS myelin-producing cells.
Progressive multifocal leukoencephalopathy (PML) occurs almost exclusively in
immunosuppressed individuals. There are only isolated case reports of PML in patients
without apparent immunosuppression [5-9]. There are, however, reports of PML
affecting patients who have conditions associated with minimal or occult
immunosuppression, such as hepatic cirrhosis and renal failure.
The typical appearance of PML on neuroimaging studies consists of symmetric or
asymmetric multifocal areas of white matter demyelination that do not conform to
cerebrovascular territories and exhibit neither mass effect nor contrast enhancement.
In patients with compatible clinical and neuroimaging features, the diagnosis of
progressive multifocal leukoencephalopathy (PML) is established by demonstrating the
presence of JC virus DNA in the cerebrospinal fluid (CSF) using polymerase chain
reaction (PCR) [76]. However, brain biopsy remains the gold standard for the diagnosis.
For patients with clinical and neuroimaging features of PML who have a negative PCR
for JC virus in the CSF, our suggested approach is to evaluate for other neurologic
disorders (see 'Differential diagnosis' below) and repeat the PCR for CSF JC virus
(algorithm 1). Brain biopsy is indicated if this evaluation and repeat PCR is unrevealing.
32. 20 yo college student follow up 2-days after head injury playing football. He did not lose
consciousness, but not remember the rest of the game. Evaluated 1h after the game in
ER. CT no abnormalities. Discharged. Today present with headache improves with
acetaminophen. No nausea, vomiting, memory problems. He asks if he can plays
football tomorrow. Recommendation?
A. No contact sport pending evaluation in 1 week.
Based on these concerns, it is recommended that
●Athletes suspected of having a concussion should be removed from play and
evaluated by a licensed health professional. An emergency department
evaluation is indicated for any athlete who suffers loss of consciousness [1,185].
(See 'Acute evaluation and management' above.)
●Athletes with diagnosed concussion should be removed from play or practice
(contact-risk activity) until symptoms have resolved off medication.
●A more conservative approach is probably appropriate for children and
adolescents. (See "Minor head trauma in infants and children: Evaluation".)
●Individuals with a history of multiple concussions should undergo a more
detailed evaluation regarding neurobehavioral symptoms; if these are present,
they should be referred for neurologic and neuropsychological assessments
[186]. Patients with persistent neurobehavioral complaints or objective deficits
should be counseled about the risk of chronic traumatic encephalopathy and
possible retirement from contact sports.
The most recently issued 2012 Consensus Statement on Concussion in Sport was written
by a multi-disciplinary, international group and proposes a six-day graduated return to
play protocol in which the athlete makes a stepwise increase in functional activity, is
evaluated for symptoms, and is allowed to progress to the next stage each successive
day if asymptomatic (table 4) [6]. If symptoms occur, then the patient should drop back
to the previous asymptomatic level and reattempt progression after 24 hours. While
these guidelines further suggest that a more rapid return to play may be possible for
asymptomatic adult athletes, same day return to play is not recommended. They also
suggest that a more conservative approach be followed for adolescents and children
Graduated return to play protocol
Rehabilitation stage Functional exercise at each stage of rehabilitation Objective of each stage
1. No activity Complete physical and cognitive rest Recovery
2. Light aerobic Walking, swimming or stationary cycling keeping intensity
Increase HR
exercise <70 percent MPHR; no resistance training
3. Sport-specific Skating drills in ice hockey, running drills in soccer; no head
Add movement
exercise impact activities
Progression to more complex training drills, eg, passing
4. Non-contact Exercise, coordination, and
drills in football and ice hockey; may start progressive
training drills cognitive load
resistance training
5. Full contact Following medical clearance, participate in normal training Restore confidence and assess
practice activities functional skills by coaching staff
6. Return to play Normal game play
Six-day return to play protocol. Each day the athlete makes a stepwise increase in functional activity, is evaluated
for symptoms, and is allowed to progress to the next stage each successive day if asymptomatic.
33. 27 yo man, e-month hx decreased libido and delayed eyaculation. Detrimental in
relationship. Not problems in the past. 4 month ago, low modd, loss interest and
productivity. Ttmt with sertraline. Normal physical examination. Neurotransmitter
responsible for symptons?
E. Serotonin
Selective serotonin reuptake inhibitors (SSRIs) can cause sexual dysfunction, including
decreased libido, decreased arousal, anorgasmia in women, and increased ejaculation
latency in men. Among patients receiving SSRIs, the estimated incidence of sexual
dysfunction is approximately 50 percent.
Our initial approach for patients who respond to SSRIs but suffer sexual dysfunction is to
wait (eg, two to eight weeks) for spontaneous remission of the sexual impairment; if the
impairment persists, we decrease the dose within the therapeutic dose range.
Patients with SSRI induced sexual dysfunction often do not respond to watchful waiting
or lower doses. For depressed patients with substantial benefit from SSRI therapy but
severe sexual dysfunction, or with modest benefit from SSRI therapy and moderate
sexual dysfunction, we suggest switching antidepressants, rather than augmenting the
SSRI with a second drug (Grade 2B). We typically switch to bupropion or mirtazapine;
however, reasonable alternatives include agomelatine, moclobemide, and selegiline, as
well as a different SSRI.
Patients treated with SSRIs who obtain substantial relief from the depressive syndrome
and suffer only moderate sexual dysfunction are typically managed by augmenting the
SSRI with a second drug, such as phosphodiesterase-5 inhibitors or bupropion. For men
with erectile dysfunction, we recommend adjunctive treatment with a
phosphodiesterase-5 inhibitor, rather than bupropion (Grade 1B). For women with
sexual dysfunction, we typically suggest add-on bupropion at higher doses rather than a
phosphodiesterase-5 inhibitor (Grade 2C). However, if female sexual dysfunction is
limited to delayed orgasm or anorgasmia, a phosphodiesterase-5 inhibitor is a
reasonable option. In addition, a reasonable alternative to augmentation for both men
and women is switching antidepressants.
34. Past 3 months, 60 yo woman with 2-min episodes rising sensation abdomen followed by
loss arousal. During episode, stares, makes chewing movements, fumbling movements
hands. Post episode confusion. Right superior quadrantanopsia without macular
sparing. Location lesion?
E. Temporal lobe
Localizing retrochiasmal visual field defects
Defect Localization Associated symptoms, signs
Homonymous sectoranopia Lateral geniculate body Optic atrophy, hemianesthesia, hemiparesis
Homonymous pie-in-the-sky Anterior temporal lobe Receptive aphasia (dominant hemisphere)
Posterior optic radiations, visual Hemisensory loss, depending on lateralization:
Homonymous quadrantanopia
cortex hemineglect Gerstmann's syndrome
Homonymous paracentral
Posterior primary visual cortex Isolated symptom
scotomas
Macular-sparing homonymous Primary visual cortex (sparing
Isolated if occipital lobe
hemianopia posterior portion)
Temporal crescent only Far anterior primary visual cortex Often isolated symptom
Temporal crescent-sparing Primary visual cortex (sparing far
Isolated symptom
homonymous hemianopia anterior portion)
Bilateral homonymous Bilateral posterior optic radiations
Anton's syndrome
hemianopia or primary visual cortex

35. 18 yo man, ER 1-hour after falling 25 feet. Paraplegic and urinary incontinence, Lower
extremities with absent reflexes and flaccid tone. Babisnky (+) bilat. X-ray, fracture
dislocation of T10. Explanation urnary incontinence?
D. Compression of the thoracic spinal cord.
Depending on the acuity, level, and completeness of the spinal cord lesion, a number of
problems can result:
●Bladder or detrusor hyperactivity produces reflexive bladder emptying.
Patients may be troubled by bladder spasms as well as urgency and frequency,
often with incontinence. Over time, this can lead to decreased capacity of the
bladder.
●Sphincter hyperactivity can impair complete emptying of the bladder.
●Detrusor sphincter dyssynergia, a combination of detrusor and sphincter
hyperactivity, can lead to bladder contractions against a closed sphincter,
leading to elevated bladder pressures and vesicoureteral reflux.
●Bladder flaccidity is produced in lower motor neuron injuries affecting the
cauda equina or conus medullaris as well as with acute upper motor neuron
injuries (spinal shock). This leads to chronic urinary retention with overflow
incontinence and incomplete emptying.
36. 67 yo woman, brought 3-hour after episode 5-min of visual loss left eye. Hx HTN and
angina. Medications lisinopril. Metoprolol. Pulse 72/min, BP 150/92. Funduscopic no
abnormalities. 3/6 holosystolic murmur at apex. No focal neurological sign. Next step
dx?
E. Carotid doplex ultrasonography.
Amaurosis fugax (from the Greek "amaurosis," meaning dark, and the Latin "fugax,"
meaning fleeting) refers to a transient loss of vision in one or both eyes [1].
Typical Associated symptoms
Etiology Pattern of visual loss Mechanism
duration and signs
Monocular ischemia,
1 to 10 Monocular, rapid onset, Hollenhorst plaque, Retinal embolism
carotid disease, other
minutes altitudinal onset hemispheric symptoms (usually)
embolic source
Headache, neck pain,
Giant Cell arteritis Variable Usually monocular Ischemia of optic nerve
jaw claudication
Monocular graying or Elevated intracranial
Papilledema Seconds Headache, diplopia
blurring pressure
Idiopathic retinal 5 to 60 Monocular positive or Transient retinal arterial
Vasospasm
vasospasm minutes negative symptoms narrowing, headache
Usually binocular, Spreading cortical
10 to 30 Usually followed by
Migraine positive symptoms with depression, possibly
minutes migraine headache
spread retinal vasospasm
Isolated or accompanied
1 to 10 Homonymous
Vertebrobasilar ischemia by other brainstem Embolic
minutes hemianopia
deficits
Binocular, lateralized,
3 to 5 Altered consciousness,
Seizure: Ictal positive phenomenon Epileptic discharge
minutes motor symptoms
common
20
Binocular visual field
Postictal minutes, Preceding ictus Cortical inhibition
loss
longer
Transient visual obscurations due to papilledema typically last seconds.
Thromboembolic events from carotid disease or elsewhere generally last 1 to 15
minutes and only rarely an hour or more [6-8]. Migraine aura typically lasts 10 to 30
minutes.
Ischemia is the most common cause of transient monocular visual loss (TMVL) [5]; this in
itself can have diverse etiologies. Papilledema, optic neuropathy, and ocular disease are
other causes of TMVL.
In contrast to transient ischemic attacks (TIAs) involving the cerebral hemispheres,
retinal ischemia is more commonly associated with emboli originating from carotid
stenosis rather than heart disease [5,28-30]. This may represent a streaming effect of
laminar blood flow, whereby particulates of uniform size are consistently deposited into
one lamina, and swept into the same distal vascular bed. Small emboli from carotid
disease may be more likely to drift to the edge of the bloodstream and enter the
ophthalmic artery. Also, these small particles may be more prone to obstruct the
smaller retinal arterioles.
Carotid imaging — Carotid duplex ultrasound, magnetic resonance angiography, or
computed tomographic angiography should be ordered in all older patients (>50
years) and in younger patients with vascular risk factors (diabetes, hypertension,
hyperlipidemia) who have experienced TMVL. Magnetic resonance imaging (MRI)
should be performed in a patient of any age with TMVL who has a history suggestive
of carotid artery dissection
GCA most often presents as acute sustained unilateral visual loss with a swollen optic
disc [36]. In two large series (each >150 patients) of biopsy-confirmed GCA, TVL was the
presenting symptom in just 10 to 15 percent [36,38]. It was bilateral in 27 to 35 percent
of these. Common accompanying symptoms include jaw claudication and headache
(each in about half of patients). However, patients with occult giant cell arteritis may
have no other symptoms [37]. Erythrocyte sedimentation rate and C-reactive protein -
All older patients (>50 years) with transient monocular or binocular vision loss should
have a sedimentation rate and C-reactive protein to exclude giant cell arteritis (GCA). If
these are elevated, or if the history is very suggestive, patients should proceed to a
confirmatory temporal artery biopsy. Treatment should not await pathology results.
37. 11 yo girl, ER 2-day hx increasing weakness and prickling sensation in her legs. Flu-like
illness past week. 98.6F, pulse 78/min, resp 20/min, BP 115/70. Lower extremities
strength 1/5, decreased sens pinprick. decreased patellar and achilles reflexes. CSF
protein 80, WBC 4. Pharmacotherapy?
E. Immune globulins
In most cases, neurologic symptoms develop two to four weeks after having what
appears to be a benign febrile respiratory or gastrointestinal infection. The predominant
symptoms of GBS at presentation in children are pain and gait difficulty. Lower
extremity symmetric weakness may ascend over hours to days to involve the arms and
the muscles of respiration in severe cases. In those with cranial neuropathy, the facial
nerve is most commonly affected, resulting in bilateral facial weakness. Autonomic
dysfunction occurs in approximately one-half of children with GBS. Physical examination
reveals symmetric weakness with diminished or absent reflexes. Most patients reach
the nadir of their function within two to four weeks, followed by return of function
occurring slowly over the course of weeks to months.
With lumbar puncture, characteristic cerebrospinal fluid findings include normal
pressure, fewer than 10 cells (typically mononuclear), and an elevated protein
concentration (>45 mg/dL).
The main modalities of therapy for GBS are intravenous immune globulin (IVIG) and
plasma exchange (also called plasmapheresis).
IVIG and plasma exchange for children with GBS should be reserved for those with any
of the following indications:
●Progressing weakness
●Worsening respiratory status or need for mechanical ventilation
●Significant bulbar weakness
●Inability to walk unaided
We prefer IVIG to plasma exchange for the treatment of children because of its relative
safety and ease of administration, although it has not been shown to have better
results. Neither treatment is recommended for ambulatory children with GBS who have
mild disease or for children whose symptoms have stabilized. However, children who
have rapid progression followed by stabilization of symptoms within the first or second
week of GBS onset may still be considered candidates for treatment.
Glucocorticoids are not beneficial for GBS and have no role in its treatment.
38. 32 yo man, ER 20 min after found in bathroom bus station. Found having continuous,
generalized tonic clonic seizures. Intubated, and supplemental O2. Flushed in
diaphoretic. No signs trauma. 99F, pulse 110/min, resp 15/min, BP 140/70. Pupils 4mm
reactive. Tonic stiffness trunk and synchronous jerking of extremities. IV dextrose
started. Next step ttmt?
B. Lorazepam

39. 25 yo woman, 6-month hx worsening excessive daytime sleepiness. Problem sustaining


attention, impair work productivity. Occasional muscles go limp and falls suddenly. BMI
22. Pulse 85/min, resp 12/min, BP 120/76. Initial step management.
E. Moldafinil therapy.
Nearly half of patients with narcolepsy report that their sleepiness and cataplexy
substantially interfere with their daily lives, including school, jobs, marriages, or social
lives [1,2]. The mainstays of therapy are brief daytime naps and pharmacologic therapy.
There are several nonpharmacologic interventions that may benefit the patient with
narcolepsy:
●Avoiding certain drugs – Some drugs should be avoided by patients with
narcolepsy. Drugs that can worsen daytime sleepiness include benzodiazepines,
opiates, antipsychotics, and alcohol. Other medications such as theophylline or
excessive caffeine can cause insomnia, which can worsen daytime sleepiness. In
addition, prazosin and other alpha-1 antagonists can worsen cataplexy.
●Napping/sleep hygiene – Behavioral interventions are often helpful for optimal
control of narcolepsy. One or two well-timed 20 minute naps will often improve
sleepiness for one to three hours though some patients only benefit from long
naps [4]. If it can be arranged, a brief nap at work or school is often helpful.
Sleep deprivation may worsen narcolepsy symptoms and therefore patients
should be counseled to maintain a regular and adequate sleep schedule [5].
●Psychosocial support – Patients with narcolepsy face various psychosocial and
work-related problems throughout their lives; as a result, they may have
difficulty meeting economic and social responsibilities [6]. They also have the
additional burden of coping with misperceptions about the causes and the
involuntary nature of their symptoms. Common misconceptions, even among
medical caregivers, are that sleep attacks and cataplexy (emotionally triggered
muscle paralysis resulting in partial or complete collapse) are manifestations of
poor motivation, denial, or avoidance. Thus, patients often benefit from
participation in support groups that focus on coping skills and identification of
community resources to assist with administrative and medical issues.
Many patients will benefit from these nonpharmacologic approaches, but most also
require medications that reduce sleepiness and cataplexy.
Modafinil has become a first line pharmacologic therapy because it provides good
control of sleepiness, is generally well tolerated, and illicit use is rare. A non-
amphetamine "wakefulness promoting agent," its mechanism of action is not well
understood, but it may increase dopaminergic signaling.
40. 68 yo woman, 3 month hx of spinning sensation. Feeling room spinning when she tries
to turn over in bed, resolves in 1 min. No vomiting, but queasy. Normal hearing. Normal
ocular movements if sitting, but coarse rotatory nystagmus to one side if head is
hanging from examination table. Abnormality in which site?
C. Otolith
Patients with benign paroxysmal positional vertigo (BPPV) present with recurrent
episodes of vertigo that last one minute or less. Episodes are provoked by specific types
of head movements, such as looking up while standing or sitting, lying down or getting
up from bed, and rolling over in bed. The vertigo may be associated with nausea and
vomiting.
Hearing loss or symptoms are typically absent [14]. Patients with BPPV typically have no
other neurologic complaints.
With the patient sitting, the neck is extended and turned to one side. The patient is then
placed supine rapidly, so that the head hangs over the edge of the bed. The patient is
kept in this position until 30 seconds has passed if no nystagmus occurs. The patient is
then returned to upright, observed for another 30 seconds for nystagmus, and the
maneuver is repeated with the head turned to the other side. A video demonstrating
this maneuver can be viewed at
http://www.neurology.org/content/70/22/2067/suppl/DC2.

Diagnostic criteria employing the Dix-Hallpike maneuver have been proposed for
posterior canal BPPV (figure 1) [18]:
●Nystagmus and vertigo usually appear with a latency of a few seconds and last
less than 30 seconds
●It has a typical trajectory, beating upward and torsionally, with the upper poles
of the eyes beating toward the ground
●After it stops and the patient sits up, the nystagmus will recur but in the
opposite direction
●The patient should then have the maneuver repeated to the same side; with
each repetition, the intensity and duration of nystagmus will diminish
Benign paroxysmal positional vertigo (BPPV) is commonly attributed to canalithiasis,
ie, calcium debris within the semicircular canal [3]. This debris likely represents loose
otoconia (calcium carbonate crystals) within the utricular sac.
The semicircular canals normally detect angular head accelerations. Heavy debris in the
canal causes inappropriate movement of the endolymph with linear accelerations, such
as gravity, and causes the erroneous sensation of spinning when the head shifts with
respect to gravity.
41. 24 yo woman, ER after 1-hour generalized tonic-clonic seizure lasted 5 min. Immigrated
from Central America 6 months ago. No medications. Drowsy but cooperative. 98.8F,
pulse 80/min, resp 18/min, BP 150/90. Toxycology (-). CT head multiple enhancing
lesions. Dx?
D. Neurocysticercosis
Neurocysticercosis was observed more frequently in emergency departments of Los
Angeles, Phoenix, and Albuquerque (5.7 percent), which had a higher proportion of
immigrant Hispanic patients than the other hospitals. Travelers to endemic areas
represent another source of cysticercosis, although such infection accounts for a
minority of cases in the United States.
The clinical manifestations depend upon whether the cysts are localized to the brain
parenchyma, the extraparenchymal tissues, or both. In general, parenchymal cysts are
associated with seizures and headache, while extraparenchymal cysts are associated
with symptoms of elevated intracranial pressure (eg, headache, nausea, and vomiting)
and may be accompanied by altered mental status. Other less common manifestations
include mass effect, altered vision, focal neurologic signs, altered mental status, and
meningitis. Fever is not typically present. Neurologic examination usually does not
demonstrate focal signs in the absence of mass effect or stroke. Parenchymal cystic or
enhancing lesions are the most common form of neurocysticercosis (NCC) in hospital-
based series and is present in >60 percent of these patients [3,12-15]. The onset of
symptomatic NCC has been estimated to peak at three to five years after infection, but
it can be delayed for >30 years [14,16,17]. Later onset may be due to calcified lesions.
Ultimately, the cystic lesions either resolve or form a calcified granuloma. When
present, the calcifications are associated with recurrent seizures, although the
mechanism of seizures associated with calcified lesions is not completely clear. Seizures
may result from inflammation, perhaps from intermittent antigen release [6,7].
Alternatively, changes in brain plasticity and scarring (eg, hippocampal sclerosis) may
result in epileptogenic foci [8,9]. Studies have highlighted a potential role of matrix
metalloproteinases in the pathogenesis of seizures in neurocysticercosis, especially in
patients with calcified lesions [10].
42. 19 yo woman, ER for seizure at dormitory party on campus. Drinking more water than
usual, then silent and staring. Shaking for 1 minute. Unresponsive, Restless and grinds
her teeth. 103.3F, pulse 90/min, resp 24/min BP 150/95. Profuse diaphoresis and
piloerection. Pupils 5mm bilat. PT 16s, Na 114, Urea nitrogen 40, Creat 2, ALT 90, CK
1200. Intoxication for which substance?
C. Ecstasy
MDMA is a commonly abused drug, particularly among young party-goers at electronic
dance music venues, including dance clubs and large music festivals. Typical effects
include feelings of euphoria, wakefulness, intimacy, sexual arousal, and disinhibition.
sympathomimetic amphetamine that causes release of endogenous catecholamines.
MDMA differs somewhat from traditional amphetamines in that it is structurally similar
to serotonin. This difference likely accounts for an increased release of serotonin and
inhibition of serotonin reuptake [4,10,11].
This results in a toxicity profile for MDMA that is different from that of traditional
amphetamines. Although typical findings of amphetamine toxicity (hypertension,
tachycardia, hyperthermia, CNS stimulation) are often seen in MDMA toxicity,
serotonergic toxicity (serotonin syndrome, SIADH) may be seen as well.
MDMA (3,4-methylenedioxymethamphetamine) increases alertness, reduces fatigue,
and leads to feelings of increased physical and mental powers, and euphoria. Users
typically begin to experience the desired effects of MDMA approximately one hour
following oral administration [10]. Minor adverse reactions, such as agitation, nausea,
bruxism (grinding teeth), ataxia, diaphoresis, blurry vision, tachycardia, and
hypertension, can also occur at typical MDMA doses. These effects are usually self-
limited and resolve within hours [14]. More serious effects are uncommon and
described below.
Vital signs — MDMA can cause hypertension, tachycardia, and hyperthermia.
Cardiovascular stimulation — Life-threatening increases in heart rate and blood
pressure may occur. Cardiovascular toxicity can include hypertensive emergencies,
myocardial infarction, aortic dissection, and dysrhythmia [15-21]. In addition,
catastrophic central neurologic hemorrhage, including intracranial hemorrhage and
posterior spinal artery aneurysm, has been reported [22,23].
Hyperthermia and related effects — Hyperthermia may result from drug effects on the
central nervous system (CNS), prolonged physical exertion (eg, dancing all night at a
"rave"), and environmental conditions (eg, dancing in a densely populated, hot room).
Both the stimulant effect of amphetamines and serotonin syndrome may contribute to
severe hyperthermia in these patients [19,24]. Hyperthermia can lead to disseminated
intravascular coagulation and rhabdomyolysis. (See 'Hyperthermia' below.)
Manifestations of hyponatremia — MDMA use can lead to hyponatremia due to a
marked increase in fluid intake and in some patients, persistent secretion of antidiuretic
hormone that slows the rate of water excretion. The belief among some MDMA users
that they can avoid hyperthermia by drinking large amounts of water is the reason for
increased fluid intake in many cases. The mechanisms that lead to hyponatremia are
described separately. (See "Causes of hyponatremia in adults", section on 'Ecstasy
(MDMA) intoxication'.)
Marked and often acute reductions in serum sodium can lead to serious neurologic
manifestations including confusion, seizures, cerebral edema, cerebral herniation, and
death. The serum sodium in such patients is usually below 120 meq/L (120 mmol/L).
Young women appear to be at increased risk for both symptomatic hyponatremia and
residual neurologic injury.
Neurologic — Stimulation of the CNS is common and can manifest as agitation,
hyperactivity, anxiety, and even delirium [24]. Seizures and status epilepticus can
occur. Psychomotor agitation may be associated with hyperthermia as well as
rhabdomyolysis.
Hepatotoxicity — Hepatotoxicity caused by MDMA poisoning is well recognized. Even in
the absence of severe hyperthermia or disseminated intravascular coagulation,
hepatitis, centrilobular necrosis, and hepatic fibrosis may result from MDMA abuse
[25,26]. Clinical findings are similar to other forms of toxin-induced hepatic injury and
may include jaundice, abdominal pain, and vomiting. Elevations of bilirubin, aspartate
transaminase (AST), and alanine transaminase (ALT) may be present.
Serotonin syndrome findings — Serotonin syndrome is a potentially life-threatening
condition characterized by the triad of autonomic dysfunction, abnormal neuromuscular
activity, and altered mental status. MDMA use can cause serotonin syndrome,
presumably via stimulation of massive serotonin release. Individuals who use MDMA in
combination with selective serotonin reuptake inhibitors (SSRIs), monoamine oxidase
inhibitors (MAOIs), or other drugs that increase 5-HT1A receptor activity (such as
meperidine, tryptophan, or lithium) are at greater risk of developing serotonin
syndrome [27-30].
43. 37 yo man, Er with 10-day hx increasingly severe headaches. Unremitting pain for 18h.
Pain is severe in pounding. Pulse 70/min, resp 16/min, BP 142/90. Mild swelling, dusky
reddish-purple skin color with venous distention over head and neck. Papilledema. No
neurological local findings. Most likely cause of intracranial HTN?
D. Obstruction of venous return to the heart
Superior vena cava (SVC) syndrome results from any condition that leads to obstruction
of blood flow through the SVC. Obstruction can be caused by invasion or external
compression of the SVC by an adjacent pathologic process involving the right lung,
lymph nodes, and other mediastinal structures, or by thrombosis of blood within the
SVC. In some cases, both external compression and thrombosis coexist.
The clinical diagnosis of superior vena cava (SVC) syndrome is made on the basis of
characteristic signs and symptoms of central venous obstruction. Regardless of etiology,
dyspnea is the most common symptom [8,9,30]. In addition, patients frequently
complain of facial swelling or head fullness, which may be exacerbated by bending
forward or lying down. Other symptoms include arm swelling, cough, chest pain, or
dysphagia. Patients with cerebral edema may have headaches, confusion, or possibly
coma.
On physical examination, the most common findings are facial edema and distension of
the veins in the neck and on the chest wall (picture 1). Arm edema, cyanosis, and facial
plethora are less frequent.
44. 52 yo man episodes of loss consciousness preceded by twitching of the right thumb,
then hand, wrist, elbow, and shoulder. Spread duration of 20 seconds. Find himself in
floor after. Bites tongue and urinates during episodes.
G. Simple partial (motor) seizure with generalization.
New Classification:
In the 2010 proposal, mode of onset is subdivided into generalized, focal, and
unknown, the latter of which includes spasms.
Generalized — Generalized seizures are conceptualized as those that originate at some
point within, and rapidly engage bilaterally distributed networks, which can be
subcortical or cortical structures. Generalized seizures do not need to necessarily
include the entire cortex, however, and they may be asymmetric. Importantly, a
generalized presentation can still arise from a focal lesion, and does not exclude the
possibility of a surgical remedy.
Focal — The term focal has replaced partial to describe seizures that originate in
networks limited to one hemisphere [3]. Focal seizures may arise from either subcortical
structures or neocortex.
In addition, the terms simple partial, complex partial, and secondarily generalized
have been eliminated, since they were difficult to define pragmatically and were often
used incorrectly. Instead, the proposal uses various descriptors of focal seizures to
describe an event more precisely. Examples of preferred descriptors include:
●Without impairment of consciousness or awareness
●Involving subjective sensory or psychic phenomena
●With impairment of consciousness or awareness, or dyscognitive
●Evolving to a bilateral convulsive seizure
Unknown — The term unknown mode of onset is used for seizure types where it
remains unclear whether onset is focal, generalized, or perhaps either. A key example is
epileptic spasms, concerning which there has traditionally been controversy, and
current knowledge is inadequate to specify mode of onset as either focal or generalized.
ILAE Classification of seizures
Generalized seizures
Tonic-clonic (in any combination)
Absence
Typical
Atypical
Absence with special features
Myoclonic absence
Eyelid myoclonia
Myoclonic
Myoclonic
Myoclonic atonic
Myoclonic tonic
Clonic
Tonic
Atonic
Focal seizures
Unknown
Epileptic spasms
ILAE: International League Against Epilepsy.
old classification:

45. 72 yo man episodes loss of consciousness past 2 years. No warming symptons, suddenly
find on the floor. Fracture arm and bruised. During episodes becomes pale and sweaty.
Jerking or twitching movements of the trunk and extremities for several seconds during
episode. He recovers completely and is not confused at the end of episodes. Hx of 2 MI.
K. Syncope
Imitators of epilepsy: Nonepileptic paroxysmal disorders
Neonates
Apnea
Jitteriness
Benign neonatal sleep myoclonus
Hyperkplexia
Infants
Breath-holding spells
Benign myoclonus of infancy
Shuddering attacks
Sandifer syndrome
Benign torticollis in infancy
Abnormal eye movements (eg, spasmus nutans, opsoclonus-myoclonus)
Rhythmic movement disorder (head banging)
Children
Breath-holding spells
Vasovagal syncope
Migraine
Benign paroxysmal vertigo
Staring spells
Tic disorders and stereotypies
Rhythmic movement disorder
Parasomnias
Adolescents and young adults
Vasovagal syncope
Narcolepsy
Periodic limb movements of sleep
Sleep starts
Paroxysmal dyskinesia
Tic disorders
Hemifacial spasm
Stiff person syndrome
Migraine
Psychogenic nonepileptic pseudoseizures
Hallucinations
Older adults
Cardiogenic syncope
Transient ischemic attack
Drop attacks
Transient global amnesia
Delirium or toxic-metabolic encephalopathy
Rapid eye movement sleep behavior disorder
Differentiation of generalized tonic-clonic seizures from pseudoseizures and syncope
Generalized seizure
Characteristic Pseudoseizure Syncope
tonic-clonic
Circumstances
Usually upright; any position if
Situation Awake or sleep Awake
cardiogenic
Sleep loss, alcohol
Emotion, injury, heat, crowds; none
Precipitating factors withdrawal, flashing Emotion
if cardiogenic
lights
Presence of others Variable Usual Variable
Motor phenomena
Vocalization At onset, if any During course None
Location of motor
Proximal limb Proximal limb None
component (if present)
Tonic; flailing; struggling or Usually atonic; if syncope lasts
Generalized motor Tonic, then clonic
thrashing, or both >20 seconds: tonic, then clonic
Partial flexion or
Tonic posture Opisthotonic -
straight
Head movements To one side or none Side to side -
Clonus/limb jerks Bilaterally synchronous Asynchronous Bilaterally synchronous
Occasional, including
Purposeful movements Absent Absent
avoidance
Biting Tongue, inside mouth Lips, arms, other people Tongue biting rare
Babinski's sign Present Absent Absent
Autonomic features
Micturition Frequent Rare Occasional
Eyes Open Closed Open
Dilated or hippus during
Pupils Normal Dilated
attacks
Colour Cyanotic or grey Rubor or normal Pale
Slow if vasovagal, weak if
Pulse Rapid, strong Normal vasodepressor; that of arrhythmias
if cardiogenic
Timing
Usual duration 1 to 5 min 5 to 60 min 1 to 2 min
Gradual; possibly sudden if
Onset Sudden Gradual
cardiogenic
Sequence of symptoms Stereotyped Variable Stereotyped
Spontaneous or induced by
Termination Spontaneous supraorbital pressure, Rapid
suggestion
Sequelae
Frequent, mild; scalp, Rare, but multiple bruises
Injury If sudden onset
face, common possible; scalp, face, rare
Regains consciousness in 2 to 3
Postictal Tired, confused, sleepy Alert, emotional outburst
min; alert but tired

Following the end of a seizure, there is a period of transition from the ictal state back to
the individual's normal level of awareness and function. This interval is referred to as
the "postictal period" and signifies the recovery period for the brain. Manifestations
typically include confusion and suppressed alertness; focal neurologic deficits may also
be present. The postictal state may last from seconds to minutes to hours, depending
upon several factors including which part(s) of the brain were affected by the seizure,
the length of the seizure, whether the individual was on antiseizure drugs, and age.
As an example, young adults with focal seizures of frontal lobe origin may have postictal
states that last only several seconds, while elderly patients with secondarily
generalized seizures may have postictal confusion and sleepiness that persists for as
long as several days to a week, particularly if there is underlying brain dysfunction
[27]. If a person had a focal seizure with impairment of consciousness or a convulsion,
his or her level of awareness gradually improves during the postictal period, much like a
person waking up from anesthesia after an operation.
46. 25 yo woman, growths in her lips and tongue for years. Nodules in lips, buccal mucosa,
tongue. Café au lait spot in trunk, limbs. Scattered nodules on skin. Greatest risk for?
D. optic nerve glioma
The typical order of appearance of clinical manifestations is café-au-lait macules, axillary
and/or inguinal freckling, Lisch nodules (iris hamartomas), and neurofibromas [32].
Osseous lesions, if present, usually appear during the patient's first year after birth, and
symptomatic optic pathway glioma (OPG) usually occurs by the time the patient is three
years of age (table 1). Other tumors and neurologic complications typically begin to
appear after the first year of life. Hypertension may occur in childhood. Malignant
transformation of tumors may also occur in childhood, but more often occurs in
adolescence and adulthood. OPGs occur in 15 percent of children younger than six years
of age with NF1 [52]. They rarely occur in older children and adults [53,54]. OPGs are
typically low-grade pilocytic astrocytomas [55]. They can arise anywhere along the
anterior visual pathway to the optic radiations and involve the optic nerves, chiasm, and
postchiasmal optic tracts.
47. 12 yo girl with SLE in ER b/c difficulty awakening this morning. Need constant
stimulation to stay awake. Past 2-day, flu-like symptoms and vomiting. Unable to take
medication corticosteroids and antihypertensive. 98.6F, pulse 60/min, resp 40/min, BP
180/130. Obtunded but moves extremities to painfull stimuli. Bilat papilledema with
flame hemorrhages. Tendon reflexes brisk symmetric. ESR 12. Dx?
C. Hypertensive encephalopathy
Hypertensive emergencies in children usually manifest as hypertensive encephalopathy:
severe BP elevation with cerebral edema and neurological symptoms of lethargy,
coma, and/or seizures. This pathology is caused by cerebrovascular endothelium
breakdown secondary to failure of cerebral autoregulation. In addition to hypertensive
encephalopathy, visual changes may also result from retinal hemorrhage or exudates
that involve the macula (central vision) or cause noticeable defects in peripheral vision.
Funduscopic examination is of particular importance since papilledema and retinal
hemorrhage or exudates may be the only sign of a hypertensive emergency.
48. 48 yo woman, hx 6-month gradual hearing loss left ear. Difficulty maintaining her
balance past 3 months. 6 months ago episode of nephrolithiasis complicated with UTI
and require antibiotic. Decreased left nasolabial fold and slight perioral droop when
smiles. Forehead normal. Gait normal, but difficulty with tamden walking. High
frequency hearing loss left with impaired speech discrimination. Dx?
A. acoustic neuroma (vestibular schwannoma)
The median age at diagnosis is approximately 50 years [2]. The tumors are unilateral in
more than 90 percent of cases [5], affecting the right and left sides with equal
frequency. Bilateral vestibular schwannomas are primarily limited to patients with
neurofibromatosis type 2. Childhood exposure to low-dose radiation for benign
conditions of the head and neck has been associated with an increased risk of vestibular
schwannoma.
The clinical presentations of these tumors are illustrated by a series of 1000 vestibular
schwannomas treated at a single institution [18]. Clinical manifestations in this series
included the following:
●Cochlear nerve — Symptomatic cochlear nerve involvement occurred in 95
percent of patients [18]. The two major symptoms were hearing loss and
tinnitus. Hearing loss was present in 95 percent but only two-thirds of these
patients were aware of this limitation. The hearing loss was usually chronic, with
an average duration of about four years. Occasionally, vestibular schwannomas
can present with sudden sensorineural hearing loss.
Tinnitus was present in 63 percent with an average duration of three years [18].
The incidence of tinnitus was higher in hearing than in deaf patients but was also
present in 46 percent of deaf patients.
●Vestibular nerve — Involvement of the vestibular nerve occurred in 61 percent
of patients [18]. Affected patients frequently acknowledged having unsteadiness
while walking, which was typically mild to moderate in nature and frequently
fluctuated in severity. True spinning vertigo was uncommon because these slow
growing tumors cause gradual rather than acute asymmetries in vestibular
function. In this setting, the central vestibular system can often compensate for
the gradual loss of input from one side.
●Trigeminal nerve — Trigeminal nerve disturbances occurred in 17 percent of
patients [18]. The most common symptoms were facial numbness (paresthesia),
hypesthesia, and pain. The average duration of symptoms was 1.3 years; the
symptoms usually occurred after hearing loss had been present for more than
two years and vestibular symptoms for more than one year.
●Facial nerve — The facial nerve was involved in 6 percent of patients [18]. The
primary symptoms were facial paresis and, less often, taste disturbances.
●Tumor progression — Other presenting signs can be the result of tumor
progression, leading to pressure on adjacent posterior fossa structures. Very
large tumors can press on the cerebellum or brainstem and result in ataxia.
Brainstem compression, cerebellar tonsil herniation, hydrocephalus and death
can occur in untreated cases. The functions of the lower cranial nerves can also
become impaired, leading to dysarthria, dysphagia, aspiration, and hoarseness.
49. 62 yo man, 5 days after hemicolectomy in ICU, present with 105F, lethargy, diaphoresis,
labile blood pressure for 3 hours. Post-op is complicated with fever, nocturnal agitation
since day 2. Haloperidol nightly (2mg) Hx of Bipolar. Medication morphine, lithium,
clonazepam. Lying in pool of sweat and drooling. 105F, pulse 140/min, BP 180/min, Wet
skin and dry mucosa. Herpetic lesion lower lip. Neck supple and IV catherer insertion
site is non-tender, no erythema. Bowel sounds hyperactive. Wound clean and dry, no
erythema. Neurological shows masked facies and intention tremor. Reflexes hyperactive
and symmetric. Babinski (-). Labs Hb 15, WBC 20000 , Neutro 80%, bands 5%, Lymphs
10%, mono 5%. Urea nitrogen 38, Creat 1.6, CK 1000. Urianalysis and CXR normal. Dx?
E. Neuroleptic malignant syndrome.
NMS is most often seen with the "typical" high potency neuroleptic agents (eg,
haloperidol, fluphenazine) [8-10]. However, every class of neuroleptic drug has been
implicated, including the low potency (eg, chlorpromazine) and the newer "atypical"
antipsychotic drugs (eg, clozapine, risperidone, olanzapine) as well as antiemetic drugs
(eg, metoclopramide, promethazine) [4,11].
NMS is defined by its association with a class of medications that block dopamine
transmission and a tetrad of distinctive clinical features: fever, rigidity, mental status
changes, and autonomic instability [3,34].
Typical symptoms — The tetrad of NMS symptoms typically evolves over one to three
days. Each feature is present in 97 to 100 percent of patients:
Mental status change is the initial symptom in 82 percent of patients [35]. It is not
surprising, given the usual psychiatric comorbidity of the typical patient, that its
significance is often underappreciated. This often takes the form of an agitated delirium
with confusion rather than psychosis. Catatonic signs and mutism can be prominent.
Evolution to profound encephalopathy with stupor and eventual coma is typical [15].
Muscular rigidity is generalized and is often extreme. The increased tone can be
demonstrated by moving the extremities and is characterized by "lead pipe rigidity" or
stable resistance through all ranges of movement. Superimposed tremor may lead to a
ratcheting quality or a cogwheel phenomenon. Other motor abnormalities include
tremor (seen in 45 to 92 percent), and less commonly, dystonia, opisthotonus, trismus,
chorea, and other dyskinesias [2,4]. Patients can also have prominent sialorrhea,
dysarthria, and dysphagia.
Hyperthermia is a defining symptom according to many diagnostic criteria.
Temperatures of more than 38ºC are typical (87 percent), but even higher
temperatures, greater than 40ºC, are common (40 percent) [4].
Autonomic instability typically takes the form of tachycardia (in 88 percent), labile or
high blood pressure (in 61 to 77 percent), and tachypnea (in 73 percent) [2,19].
Dysrhythmias may occur. Diaphoresis is often profuse.
In an analysis of 340 cases, 70 percent of patients followed a typical course of mental
status changes appearing first, followed by rigidity, then hyperthermia, and autonomic
dysfunction [35]. Some case reports document delay in the appearance of fever of more
than 24 hours, leading to initial diagnostic confusion [2]. There is substantial variability
in the presentation of NMS, and other reports do not necessarily substantiate a typical
course.
Drugs that can cause neuroleptic malignant syndrome
Neuroleptic agents Antiemetic agents
Aripiprazole Domperidone
Chlorpromazine Droperidol
Clozapine Metoclopromide
Fluphenazine Prochlorperazine
Haloperidol Promethazine
Olanzapine
Paliperidone
Perphenazine
Quetiapine
Risperidone
Thioridazine
Ziprasidone
50. 62 yo woman, week hx intermittent drooping of his left eyelid and double vision.
Bilateral ptosis, more prominent on left. Weaknees on left eye abduction/adduction and
right abduction. Pupillary reflexes normal. Greatest risk for?
E. Thymoma
In adults, thymomas and thymic carcinomas are the most common neoplasms arising in
the thymus. Thymomas account for about 20 percent of mediastinal neoplasms (table
1). Most thymoma patients are between 40 and 60 years of age, and there is a similar
incidence in men and women [1]. There are no known risk factors, although there is a
strong association with myasthenia gravis and other paraneoplastic syndromes.
The most common paraneoplastic syndrome associated with thymoma is myasthenia
gravis. Myasthenia gravis is an autoimmune disorder caused by interference with
acetylcholine receptors of voluntary muscle at the neuromuscular junction. Common
symptoms include diplopia, ptosis, dysphagia, weakness, and fatigue.
Paraneoplastic syndromes associated with thymic neoplasms
Neurologic and Myasthenia gravis, polymyositis, sensory neuropathy, stiff person syndrome, neuromyelitis
neuromuscular optica, Isaac's syndrome (neuromyotonia), Eaton Lambert syndrome, hemichorea
Hematologic Pure red cell aplasia, agranulocytosis, hemolytic anemia, pernicious anemia
Dermatologic Alopecia areata, pemphigus, scleroderma, oral lichen planus, vitiligo
Endocrine Addison's disease, Cushing syndrome, panhypopituitarism, thyroiditis
Acquired hypogammaglobulinemia, myocarditis, nephrotic syndrome, rheumatoid arthritis,
Miscellaneous
sarcoidosis, hepatitis, gastrointestinal pseudoobstruction, ulcerative colitis

The clinical features of Lambert-Eaton myasthenic syndrome LEMS were well defined
by Lambert and Eaton in Minnesota [29] and by O'Neill and colleagues in England [18].
Most patients present with complaints of slowly progressive proximal muscle weakness,
and this feature is present in almost all patients at some point in the illness. Occasional
patients have a subacute or acute presentation.
Patients typically describe an alteration in gait or difficulty arising from a chair or
managing stairs. Many describe aching or stiff muscles. Muscle fatigability or cramping is
common, particularly after protracted exercise. There tends to be relative preservation
of distal muscle function.
Symmetrical muscle involvement is the rule. Upper extremity complaints are usually
modest and typically involve proximal muscle function. Patients with LEMS and small
cell lung cancer typically develop a more rapidly progressive course of proximal and
then distal arm muscle weakness than patients who have non-tumor LEMS
Autonomic dysfunction is often present and can be an important clue to the diagnosis.
Dry mouth from reduced salivation is the most common autonomic symptom and
occasionally is the presenting complaint, while erectile dysfunction is common in men
with LEMS. In the largest study of autonomic dysfunction in LEMS involving 30 patients,
dry mouth was identified in 77 percent, and impotence was present in 45 percent of all
men [31]. Other symptoms may include blurred vision and constipation.

You might also like